Как найти площадь трапеции если все стороны известны: Все формулы площади трапеции. Найти онлайн

Содержание

Площадь трапеции по сторонам | Треугольники

Как найти площадь трапеции по 4 сторонам?

Чтобы найти площадь трапеции, нужно знать её основания и высоту. Основания известны, следовательно, задача сводится к нахождению высоты трапеции.

I способ.

Из вершины тупого угла провести прямую, параллельную боковой стороне.

Найти площадь полученного треугольника по формуле Герона. Зная площадь, найти высоту треугольника, которая является также высотой трапеции.

Задача 1.

Найти площадь трапеции, основания которой равны 11 см и 28 см, а боковые стороны — 25 см и 26 см.

Дано: ABCD — трапеция,

AD∥BC, AB=25 см, BC=11 см,

CD=26 см, AD=28 см

Найти:

   

Решение:

1) Проведем через вершину C прямую CL,  CL∥AB.

Четырехугольник ABCL — параллелограмм (по определению, так как BC∥AL — по условию, CL∥AB — по построению).

По свойству параллелограмма, AL=BC=11 см, CL=AB=25 см. Следовательно, LD=AD-AL=28-11=17 см.

2) Рассмотрим треугольник CDL. Его площадь найдём по формуле Герона

   

   

   

   

   

С другой стороны, 

   

   

3) По формуле

   

найдём площадь трапеции ABCD:

   

   

Ответ: 468 см².

II способ.

Провести из тупых углов трапеции две высоты.

В результате получим прямоугольник и два прямоугольных треугольника.

Один из катетов этих треугольников — высота трапеции. Её можно выразить через другие стороны в каждом из треугольников, затем приравнять полученные равенства.

Задача 2.

Найти площадь трапеции, основания которой равны 10см  и 14 см, а боковые стороны — 13 см и 14 см.

рисунок 1

Дано:ABCD — трапеция,

AD∥BC, AB=13 см, BC=10 см,

CD=15 см, AD=14 см

Найти:

   

Решение:

Проведём высоты трапеции BK и CF.

Четырёхугольник BCFK — прямоугольник (так как у него все углы прямые). Поэтому, KF=BC=10 см.

Пусть FD=x см, тогда AK=AD-KF-FD=14-10-x=4-x см.

Рассмотрим треугольник CDF — прямоугольный. По теореме Пифагора

   

   

Аналогично, из треугольника ABK

   

   

Приравниваем правые части:

   

   

   

   

   

   

   

Ответ: 144 см².

рисунок 2

Традиционно трапецию изображают именно в таком виде, как на рисунке 1 — с двумя тупыми углами при меньшем основании.

Но в трапеции также могут быть тупыми противоположные углы — как на рисунке 2.

 

 

Для  трапеции с противоположными тупыми углами верны все рассуждения, приведенные выше, за одним исключением —  в этом случае BC=AF=AK+AF.

В разных вариантах трапеции отрезки FD и AK имеют разную длину, но величина высоты, а значит, и площади, одинакова.

Посчитать площадь трапеции по четырем сторонам онлайн. Как найти площадь трапеции: формулы и примеры

Для того чтобы чувствовать себя на уроках геометрии уверенно и успешно решать задачи, недостаточно выучить формулы. Их нужно в первую очередь понимать. Бояться, а тем более ненавидеть формулы — непродуктивно. В этой статье доступным языком будут проанализированы различные способы поиска площади трапеции. Для лучшего усвоения соответствующих правил и теорем уделим некоторое внимание ее свойствам. Это поможет разобраться в том, как работают правила и в каких случаях следует применять те или иные формулы.

Определяем трапецию

Что это за фигура в целом? Трапецией называют многоугольник из четырех углов с двумя параллельными сторонами. Две другие стороны трапеции могут быть наклонены под различными углами. Ее параллельные стороны называют основаниями, а для непараллельных сторон применяют наименование «боковые стороны» или «бедра». Такие фигуры довольно часто встречаются в обыденной жизни. Контуры трапеции можно увидеть в силуэтах одежды, предметах интерьера, мебели, посуды и многих других. Трапеция бывает разных видов: разносторонняя, равнобокая и прямоугольная. Более детально их типы и свойства разберем далее в статье.

Свойства трапеции

Остановимся коротко на свойствах этой фигуры. Сумма углов, прилегающих к любой боковой стороне, всегда равняется 180°. Надо заметить, что все углы трапеции в сумме составляют 360°. У трапеции существует понятие средней линии. Если соединить середины боковых сторон отрезком — это и будет средняя линия. Ее обозначают m. У средней линии есть важные свойства: она всегда параллельна основаниям (мы помним, что основания также параллельны между собой) и равна их полусумме:

Это определение обязательно надо выучить и понять, ведь это ключ к решению множества задач!

У трапеции всегда можно опустить высоту на основание. Высота — это перпендикуляр, часто обозначаемый символом h, который проведен из любой точки одного основания на другое основание или его продолжение. Средняя линия и высота помогут найти площадь трапеции. Подобные задачи являются самыми распространенными в школьном курсе геометрии и регулярно появляются среди контрольных и экзаменационных работ.

Самые простые формулы площади трапеции

Разберем две самые популярные и простые формулы, с помощью которых находят площадь трапеции. Достаточно умножить высоту на полусумму оснований, чтобы легко найти искомое:

S = h*(a + b)/2.

В этой формуле a, b обозначают основания трапеции, h — высоту. Для удобства восприятия в этой статье знаки умножения отмечены символом (*) в формулах, хотя в официальных справочниках знак умножения обычно опускают.

Рассмотрим пример.

Дано: трапеция с двумя основаниями, равными 10 и 14 см, высота составляет 7 см. Чему равна площадь трапеции?

Разберем решение этой задачи. По этой формуле сначала нужно найти полусумму оснований: (10+14)/2 = 12. Итак, полусумма равняется 12 см. Теперь полусумму умножаем на высоту: 12*7 = 84. Искомое найдено. Ответ: площадь трапеции равна 84 кв. см.

Вторая известная формула гласит: площадь трапеции равна произведению средней линии на высоту трапеции. То есть фактически вытекает из предшествующего понятия средней линии: S=m*h.

Использование диагоналей для вычислений

Другой способ нахождения площади трапеции на самом деле не так уж сложен. Он связан с ее диагоналями. По этой формуле для нахождения площади требуется умножить полупроизведение ее диагоналей (d 1 d 2) на синус угла между ними:

S = ½ d 1 d 2 sina.

Рассмотрим задачу, которая показывает применение этого способа. Дано: трапеция с длиной диагоналей равной соответственно 8 и 13 см. Угол a между диагоналями равняется 30°. Найти площадь трапеции.

Решение. Используя вышеприведенную формулу, легко вычислить требуемое. Как известно, sin 30° составляет 0,5. Следовательно, S = 8*13*0,5=52. Ответ: площадь равна 52 кв. см.

Ищем площадь равнобокой трапеции

Трапеция может быть равнобокой (равнобедренной). Ее боковые стороны одинаковы И углы при основаниях равны, что хорошо иллюстрирует рисунок. Равнобедренная трапеция имеет такие же свойства, что и обычная, плюс ряд особых. Вокруг равнобокой трапеции может быть описана окружность, и в нее может быть вписана окружность.

Какие же есть методики вычисления площади такой фигуры? Нижеприведенный способ потребует больших вычислений. Для его применения нужно знать значения синуса (sin) и косинуса (cos) угла при основании трапеции. Для их расчетов требуются либо таблицы Брадиса либо инженерный калькулятор. Вот эта формула:

S = c *sin a *(a c *cos a ),

где с — боковое бедро, a — угол при нижнем основании.

Равнобокая трапеция обладает диагоналями одинаковой длины. Верно и обратное утверждение: если у трапеции диагонали равны, то она является равнобедренной. Отсюда следующая формула, помогающая найти площадь трапеции — полупроизведение квадрата диагоналей на синус угла между ними: S = ½ d 2 sina.

Находим площадь прямоугольной трапеции

Известен частный случай прямоугольной трапеции. Это трапеция, у которой одна боковая сторона (ее бедро) примыкает к основаниям под прямым углом. Она имеет свойства обычной трапеции. Помимо этого, она обладает очень интересной особенностью. Разность квадратов диагоналей такой трапеции равняется разности квадратов ее оснований. Для нее используют все ранее приведенные методики вычисления площади.

Применяем смекалку

Есть одна хитрость, которая может помочь в случае забывчивости специфических формул. Рассмотрим внимательнее, что представляет собой трапеция. Если мысленно разделить ее на части, то мы получим знакомые и понятные геометрические фигуры: квадрат или прямоугольник и треугольник (один или два). Если известны высота и стороны трапеции, можно воспользоваться формулами площади треугольника и прямоугольника, после чего сложить все полученные величины.

Проиллюстрируем это следующим примером. Дана прямоугольная трапеция. Угол C = 45°, углы A, D составляют 90°. Верхнее основание трапеции равно 20 см, высота равна 16 см. Требуется вычислить площадь фигуры.

Данная фигура очевидным образом состоит из прямоугольника (если два угла равны 90°) и треугольника. Так как трапеция прямоугольная, следовательно, ее высота равна ее боковой стороне, то есть 16 см. Имеем прямоугольник со сторонами 20 и 16 см соответственно. Рассмотрим теперь треугольник, угол которого равен 45°. Мы знаем, что одна его сторона составляет 16 см. Так как эта сторона является одновременно высотой трапеции (а нам известно, что высота опускается на основание под прямым углом), следовательно, второй угол треугольника равен 90°. Отсюда оставшийся угол треугольника составляет 45°. Следствием этого мы получаем прямоугольный равнобедренный треугольник, у которого две стороны одинаковы. Значит, другая сторона треугольника равна высоте, то есть 16 см. Осталось вычислить площадь треугольника и прямоугольника и сложить полученные величины.

Площадь прямоугольного треугольника равна половине произведения его катетов: S = (16*16)/2 = 128. Площадь прямоугольника равняется произведению его ширины на длину: S = 20*16 = 320. Мы нашли требуемое: площадь трапеции S = 128 + 320 = 448 кв. см. Можно легко себя перепроверить, воспользовавшись вышеприведенными формулами, ответ будет идентичен.

Используем формулу Пика


Напоследок приведем еще одну оригинальную формулу, помогающую искать площадь трапеции. Она называется формулой Пика. Ею удобно пользоваться, когда трапеция нарисована на клетчатой бумаге. Подобные задачи часто встречаются в материалах ГИА. Выглядит она следующим образом:

S = M/2 + N — 1,

в этой формуле M — количество узлов, т.е. пересечений линий фигуры с линиями клетки на границах трапеции (оранжевые точки на рисунке), N — количество узлов внутри фигуры (синие точки). Удобнее всего пользоваться ею при нахождении площади неправильного многоугольника. Тем не менее, чем больше арсенал используемых методик, тем меньше ошибок и лучше результаты.

Разумеется, приведенными сведениями далеко не исчерпываются типы и свойства трапеции, а также способы поиска ее площади. В этой статье дан обзор наиболее важных ее характеристик. В решении геометрических задач важно действовать постепенно, начинать с легких формул и задач, последовательно закреплять понимание, переходить на другой уровень сложности.

Собранные воедино самые распространенные формулы помогут ученикам сориентироваться в разнообразных способах вычисления площади трапеции и более качественно подготовиться к тестам и контрольным работам по этой теме.

В математике известно несколько видов четырехугольников: квадрат, прямоугольник, ромб, параллелограмм. Среди них и трапеция — вид выпуклого четырехугольника, у которого две стороны параллельны, а две другие нет. Параллельные противоположные стороны называются основаниями, а две другие – боковыми сторонами трапеции. Отрезок, который соединяет середины боковых сторон, называется средней линией. Существует несколько видов трапеций: равнобедренная, прямоугольная, криволинейная. Для каждого вида трапеции есть формулы для нахождения площади.

Площадь трапеции

Чтобы найти площадь трапеции, нужно знать длину ее оснований и высоту. Высота трапеции — это отрезок, перпендикулярный основаниям. Пусть верхнее основание — a, нижнее основание — b, а высота — h. Тогда вычислить площадь S можно по формуле:

S = ½ * (a+b) * h

т. е. взять полусумму оснований, умноженную на высоту.

Также удастся вычислить площадь трапеции, если известно значение высоты и средней линии. Обозначим среднюю линию — m. Тогда

Решим задачу посложнее: известны длины четырех сторон трапеции — a, b, c, d. Тогда площадь отыщется по формуле:


Если известны длины диагоналей и угол между ними, то площадь ищется так:

S = ½ * d1 * d2 * sin α

где d с индексами 1 и 2 — диагонали. В данной формуле в расчете приводится синус угла.

При известных длинах оснований a и b и двух углах при нижнем основании площадь вычисляется так:

S = ½ * (b2 — a2) * (sin α * sin β / sin(α + β))

Площадь равнобедренной трапеции

Равнобедренная трапеция — это частный случай трапеции. Ее отличие в том, что такая трапеция — это выпуклый четырехугольник с осью симметрии, проходящей через середины двух противоположных сторон. Ее боковые стороны равны.


Найти площадь равнобедренной трапеции можно несколькими способами.

  • Через длины трех сторон. В этом случае длины боковых сторон будут совпадать, поэтому обозначены одной величиной — с, а и b — длины оснований:

  • Если известна длина верхнего основания, боковой стороны и величина угла при нижнем основании, то площадь вычисляется так:

S = c * sin α * (a + c * cos α)

где а — верхнее основание, с — боковая сторона.

  • Если вместо верхнего основания известна длина нижнего – b, площадь рассчитывается по формуле:

S = c * sin α * (b – c * cos α)

  • Если когда известны два основания и угол при нижнем основании, площадь вычисляется через тангенс угла:

S = ½ * (b2 – a2) * tg α

  • Также площадь рассчитывается через диагонали и угол между ними. В этом случае диагонали по длине равны, поэтому каждую обозначаем буквой d без индексов:

S = ½ * d2 * sin α

  • Вычислим площадь трапеции, зная длину боковой стороны, средней линии и величину угла при нижнем основании.

Пусть боковая сторона — с, средняя линия — m, угол — a, тогда:

S = m * c * sin α

Иногда в равностороннюю трапецию можно вписать окружность, радиус которой будет — r.


Известно, что в любую трапецию можно вписать окружность, если сумма длин оснований равна сумме длин ее боковых сторон. Тогда площадь найдется через радиус вписанной окружности и угол при нижнем основании:

S = 4r2 / sin α

Такой же расчет производится и через диаметр D вписанной окружности (кстати, он совпадает с высотой трапеции):

Зная основания и угол, площадь равнобедренной трапеции вычисляется так:

S = a * b / sin α

(эта и последующие формулы верны только для трапеций с вписанной окружностью).


Через основания и радиус окружности площадь ищется так:

Если известны только основания, то площадь считается по формуле:


Через основания и боковую линию площадь трапеции с вписанным кругом и через основания и среднюю линию — m вычисляется так:

Площадь прямоугольной трапеции

Прямоугольной называется трапеция, у которой одна из боковых сторон перпендикулярна основаниям. В этом случае боковая сторона по длине совпадает с высотой трапеции.

Прямоугольная трапеция представляет из себя квадрат и треугольник. Найдя площадь каждой из фигур, сложите полученные результаты и получите общую площадь фигуры.


Также для вычисления площади прямоугольной трапеции подходят общие формулы для расчета площади трапеции.

  • Если известны длины оснований и высота (или перпендикулярная боковая сторона), то площадь рассчитывается по формуле:

S = (a + b) * h / 2

В качестве h (высоты) может выступать боковая сторона с. Тогда формула выглядит так:

S = (a + b) * c / 2

  • Другой способ рассчитать площадь — перемножить длину средней линии на высоту:

или на длину боковой перпендикулярной стороны:

  • Следующий способ вычисления — через половину произведения диагоналей и синус угла между ними:

S = ½ * d1 * d2 * sin α


Если диагонали перпендикулярны, то формула упрощается до:

S = ½ * d1 * d2

  • Еще один способ вычисления — через полупериметр (сумма длин двух противоположных сторон) и радиус вписанной окружности.

Эта формула действительна для оснований. Если брать длины боковых сторон, то одна из них будет равна удвоенному радиусу. Формула будет выглядеть так:

S = (2r + c) * r

  • Если в трапецию вписана окружность, то площадь вычисляется так же:

где m — длина средней линии.

Площадь криволинейной трапеции

Криволинейная трапеция представляет из себя плоскую фигуру, ограниченную графиком неотрицательной непрерывной функции y = f(x), определенной на отрезке , осью абсцисс и прямыми x = a, x = b. По сути, две ее стороны параллельны друг другу (основания), третья сторона перпендикулярна основаниям, а четвертая представляет из себя кривую, соответствующую графику функции.


Площадь криволинейной трапеции ищут через интеграл по формуле Ньютона-Лейбница:


Так вычисляются площади различных видов трапеций. Но, помимо свойств сторон, трапеции обладают одинаковыми свойствами углов. Как у всех существующих четырехугольников, сумма внутренних углов трапеции равна 360 градусов. А сумма углов, прилежащих к боковой стороне, — 180 градусам.

Существует множество способов найти площадь трапеции. Обычно репетитор по математике владеет несколькими приемами ее вычисления, остановимся на них подробнее:
1) , где AD и BC основания, а BH-высота трапеции. Доказательство: проведем диагональ BD и выразим площади треугольников ABD и CDB через полупроизведение их оснований на высоту:

, где DP – внешняя высота в

Сложим почленно эти равенства и учитывая, что высоты BH и DP равны, получим:

Вынесем за скобку

Что и требовалось доказать.

Следствие из формулы площади трапеции:
Так как полусумма оснований равна MN — средней линии трапеции, то

2) Применение общей формулы площади четырехугольника .
Площадь четырехугольника равна половине произведения диагоналей, умноженной на синус угла между ними
Для доказательства достаточно разбить трапецию на 4 треугольника, выразить площадь каждого через «половину произведения диагоналей на синус угла между ними» (в качестве угла берется , сложить получившиеся выражения, вынести за скобку и раскладываю эту скобку на множители методом группировки получить ее равенство выражению . Отсюда

3) Метод сдвига диагонали
Это мое название. В школьных учебниках репетитор по математике не встретит такого заголовка. Описание приема можно найти только в дополнительных учебных пособиях в качестве примера решения какой-нибудь задачи. Отмечу, что большинство интересных и полезных фактов планиметрии репетиторы по математике открывают ученикам в процессе выполнения практической работы. Это крайне неоптимально, ибо школьнику нужно выделять их в отдельные теоремы и называть «громкими именами». Одно из таких – «сдвиг диагонали». О чем идет речь? Проведем через вершину B прямую параллельную к АС до пересечения с нижним основанием в точке E. В таком случае четырехугольник EBCA будет параллелограммом (по определению) и поэтому BC=EA и EB=AC. Нам сейчас важно первое равенство. Имеем:

Заметим, что треугольник BED, площадь которого равна площади трапеции, имеет еще несколько замечательных свойств:
1) Его площадь равна площади трапеции
2) Его равнобедренность происходит одновременно с равнобедренность самой трапеции
3) Верхний его угол при вершине B равен углу между диагоналями трапеции (что очень часто используется в задачах)
4) Его медиана BK равна расстоянию QS между серединами оснований трапеции. С применением этого свойства я недавно столкнулся при подготовке ученика на мехмат МГУ по учебнику Ткачука, вариант 1973 года (задача приводится внизу страницы).

Спецприемы репетитора по математике.

Иногда я предлагаю задачи на весьма хитрый путь нахождении я площади трапеции. Я отношу его к спецприемам ибо на практике репетитор их использует крайне редко. Если вам нужна подготовка к ЕГЭ по математике только в части B, можно про них и не читать. Для остальных рассказываю дальше. Оказывается площадь трапеции в два раза больше площади треугольника с вершинами в концах одной боковой стороны и серединой другой, то есть треугольника ABS на рисунке:
Доказательство: проведем высоты SM и SN в треугольниках BCS и ADS и выразим сумму площадей этих треугольников:

Так как точка S – середина CD, то (докажите это сами).Найдем cумму площадей треугольников:

Так как эта сумма оказалась равной половине площади трапеции, то — вторая ее половина. Ч.т.д.

В копилку спецприемов репетитора я бы отнес форму вычисления площади равнобедренной трапеции по ее сторонам: где p – полупериметр трапеции. Доказательство я приводить не буду. Иначе ваш репетитор по математике останется без работы:). Приходите на занятия!

Задачи на площадь трапеции:

Замечание репетитора по математике : Нижеприведенный список не является методическим сопровождением к теме, это только небольшая подборка интересных задач на вышерассмотренные приемы.

1) Нижнее основание равнобедренной трапеции равно 13, а верхнее равно 5. Найдите площадь трапеции, если ее диагональ перпендикулярна боковой стороне.
2) Найдите площадь трапеции, если ее основания равны 2см и 5см, а боковые стороны 2см и 3см.
3) В равнобокой трапеции большее основание равно 11, боковая сторона равна 5, а диагональ равна Найти площадь трапеции.
4) Диагональ равнобокой трапеции равна 5, а средняя линия равна 4. Найти площадь.
5) В равнобедренной трапеции основания равны 12 и 20, а диагонали взаимно перпендикулярны. Вычислить площадь трапеции
6) Диагональ равнобокой трапеции составляет с ее нижним основанием угол . Найти площадь трапеции, если ее высота равна 6см.
7) Площадь трапеции равна 20, а одна из ее боковых сторон равна 4 см. Найдите расстояние до нее от середины противоположной боковой стороны.
8) Диагональ равнобокой трапеции делит ее на треугольники с площадями 6 и 14. Найти высоту, если боковая сторона равна 4.
9) В трапеции диагонали равны 3 и 5, а отрезок, соединяющий середины оснований равен 2. Найти площадь трапеции (Мехмат МГУ, 1970г).

Я выбирал не самые сложные задачи (не стоит пугаться мехмата!) с расчетом на возможность их самостоятельного решения. Решайте на здоровье! Если вам нужна подготовка к ЕГЭ по математике, то без участия в этом процессе формулы площади трапеции могут возникнуть серьезные проблемы даже с задачей B6 и тем более с C4. Не запускайте тему и в случае каких-либо затруднений обращайтесь за помощью. Репетитор по математике всегда рад вам помочь.

Колпаков А.Н.
Репетитор по математике в Москве , подготовка к ЕГЭ в Строгино .

Многоликая трапеция… Она может быть произвольной, равнобедренной или прямоугольной. И в каждом случае нужно знать, как найти площадь трапеции. Конечно, проще всего запомнить основные формулы. Но иногда проще воспользоваться той, которая выведена с учетом всех особенностей конкретной геометрической фигуры.

Несколько слов о трапеции и ее элементах

Любой четырехугольник, у которого две стороны параллельны, можно назвать трапецией. В общем случае они не равны и называются основаниями. Большее из них — нижнее, а другое — верхнее.

Две другие стороны оказываются боковыми. У произвольной трапеции они имеют различную длину. Если же они равны, то фигура становится равнобедренной.

Если вдруг угол между любой боковой стороной и основанием окажется равным 90 градусам, то трапеция является прямоугольной.

Все эти особенности могут помочь в решении задачи о том, как найти площадь трапеции.

Среди элементов фигуры, которые могут оказаться незаменимыми в решении задач, можно выделить такие:

  • высота, то есть отрезок, перпендикулярный обоим основаниям;
  • средняя линия, которая имеет своими концами середины боковых сторон.

По какой формуле вычислить площадь, если известны основания и высота?

Это выражение дается основным, потому что чаще всего можно узнать эти величины, даже когда они не даны явно. Итак, чтобы понять, как найти площадь трапеции, потребуется сложить оба основания и разделить их на два. Получившееся значение потом еще умножить на значение высоты.

Если обозначить основания буквами а 1 и а 2 , высоту — н, то формула для площади будет выглядеть так:

S = ((а 1 + а 2)/2)*н.

Формула, по которой вычисляется площадь, если даны ее высота и средняя линия

Если посмотреть внимательно на предыдущую формулу, то легко заметить, что в ней явно присутствует значение средней линии. А именно, сумма оснований, деленная на два. Пусть средняя линия будет обозначена буквой l, тогда формула для площади станет такой:

S = l * н.

Возможность найти площадь по диагоналям

Этот способ поможет, если известен угол, образованный ими. Предположим, что диагонали обозначены буквами д 1 и д 2 , а углы между ними — α и β. Тогда формула того, как найти площадь трапеции, будет записана следующим образом:

S = ((д 1 * д 2)/2) * sin α.

В этом выражении можно легко заменить α на β. Результат не изменится.

Как узнать площадь, если известны все стороны фигуры?

Бывают и такие ситуации, когда в этой фигуре известны именно стороны. Эта формула получается громоздкой и ее сложно запомнить. Но возможно. Пусть боковые стороны имеют обозначение: в 1 и в 2 , основание а 1 больше, чем а 2 . Тогда формула площади примет такой вид:

S = ((а 1 + а 2) / 2) * √ {в 1 2 — [(а 1 — а 2) 2 + в 1 2 — в 2 2) / (2 * (а 1 — а 2))] 2 }.

Способы вычисления площади равнобедренной трапеции

Первый связан с тем, что в нее можно вписать окружность. И, зная ее радиус (он обозначается буквой r), а также угол при основании — γ, можно воспользоваться такой формулой:

S = (4 * r 2) / sin γ.

Последняя общая формула, которая основана на знании всех сторон фигуры, существенно упростится за счет того, что боковые стороны имеют одинаковое значение:

S = ((а 1 + а 2) / 2) * √ {в 2 — [(а 1 — а 2) 2 / (2 * (а 1 — а 2))] 2 }.

Методы вычисления площади прямоугольной трапеции

Понятно, что подойдет любой из перечисленных для произвольной фигуры. Но иногда полезно знать об одной особенности такой трапеции. Она заключается в том, что разность квадратов длин диагоналей равна разности, составленной из квадратов оснований.

Часто формулы для трапеции забываются, в то время как выражения для площадей прямоугольника и треугольника помнятся. Тогда можно применить простой способ. Разделить трапецию на две фигуры, если она прямоугольная, или три. Одна точно будет прямоугольником, а вторая, или две оставшиеся, треугольниками. После вычисления площадей этих фигур останется их только сложить.

Это достаточно простой способ того, как найти площадь прямоугольной трапеции.

Как быть, если известны координаты вершин трапеции?

В этом случае потребуется воспользоваться выражением, которое позволяет определить расстояние между точками. Его можно применить три раза: для того, чтобы узнать оба основания и одну высоту. А потом просто применить первую формулу, которая описана немного выше.

Для иллюстрации такого метода можно привести такой пример. Даны вершины с координатами А(5; 7), В(8; 7), С(10; 1), Д(1; 1). Нужно узнать площадь фигуры.

До того как найти площадь трапеции, по координатам нужно вычислить длины оснований. Потребуется такая формула:

длина отрезка = √{(разность первых координат точек) 2 + (разность вторых координат точек) 2 }.

Верхнее основание обозначено АВ, значит, его длина будет равна √{(8-5) 2 + (7-7) 2 } = √9 = 3. Нижнее — СД = √ {(10-1) 2 + (1-1) 2 } = √81 = 9.

Теперь нужно провести высоту из вершины на основание. Пусть ее начало будет в точке А. Конец отрезка окажется на нижнем основании в точке с координатами (5; 1), пусть это будет точка Н. Длина отрезка АН получится равной √{(5-5) 2 + (7-1) 2 } = √36 = 6.

Осталось только подставить получавшиеся значения в формулу площади трапеции:

S = ((3 + 9) / 2) * 6 = 36.

Задача решена без единиц измерения, потому что не указан масштаб координатной сетки. Он может быть как миллиметр, так и метр.

Примеры задач

№ 1. Условие. Известен угол между диагоналями произвольной трапеции, он равен 30 градусам. Меньшая диагональ имеет значение 3 дм, а вторая больше ее в 2 раза. Необходимо посчитать площадь трапеции.

Решение. Для начала нужно узнать длину второй диагонали, потому что без этого не удастся сосчитать ответ. Вычислить ее несложно, 3 * 2 = 6 (дм).

Теперь нужно воспользоваться подходящей формулой для площади:

S = ((3 * 6) / 2) * sin 30º = 18/2 * ½ = 4,5 (дм 2). Задача решена.

Ответ: площадь трапеции равна 4,5 дм 2 .

№ 2. Условие. В трапеции АВСД основаниями являются отрезки АД и ВС. Точка Е — середина стороны СД. Из нее проведен перпендикуляр к прямой АВ, конец этого отрезка обозначен буквой Н. Известно, что длины АВ и ЕН равны соответственно 5 и 4 см. Нужно вычислить площадь трапеции.

Решение. Для начала нужно сделать чертеж. Поскольку значение перпендикуляра меньше стороны, к которой он проведен, то трапеция будет немного вытянутой вверх. Так ЕН окажется внутри фигуры.

Чтобы отчетливо увидеть ход решения задачи, потребуется выполнить дополнительное построение. А именно, провести прямую, которая будет параллельна стороне АВ. Точки пересечения этой прямой с АД — Р, а с продолжением ВС — Х. Получившаяся фигура ВХРА — параллелограмм. Причем его площадь равна искомой. Это связано с тем, что треугольники, которые получились при дополнительном построении, равны. Это следует из равенства стороны и двух прилежащих к ней углов, один — вертикальный, другой — накрест лежащий.

Найти площадь параллелограмма можно по формуле, которая содержит произведение стороны и высоты, опущенной на нее.

Таким образом, площадь трапеции равна 5 * 4 = 20 см 2 .

Ответ: S = 20 см 2 .

№ 3. Условие. Элементы равнобедренной трапеции имеют такие значения: нижнее основание — 14 см, верхнее — 4 см, острый угол — 45º. Нужно вычислить ее площадь.

Решение. Пусть меньшее основание имеет обозначение ВС. Высота, проведенная из точки В, будет называться ВН. Поскольку угол 45º, то треугольник АВН получится прямоугольный и равнобедренный. Значит, АН=ВН. Причем АН очень легко найти. Она равна половине разности оснований. То есть (14 — 4) / 2 = 10 / 2 = 5 (см).

Основания известны, высота сосчитана. Можно пользоваться первой формулой, которая здесь была рассмотрена для произвольной трапеции.

S = ((14 + 4) / 2) * 5 = 18/2 * 5 = 9 * 5 = 45 (см 2).

Ответ: Искомая площадь равна 45 см 2 .

№ 4. Условие. Имеется произвольная трапеция АВСД. На ее боковых сторонах взяты точки О и Е, так что ОЕ параллельна основанию АД. Площадь трапеции АОЕД в пять раз больше, чем у ОВСЕ. Вычислить значение ОЕ, если известны длины оснований.

Решение. Потребуется провести две параллельные АВ прямые: первую через точку С, ее пересечение с ОЕ — точка Т; вторую через Е и точкой пересечения с АД будет М.

Пусть неизвестная ОЕ=х. Высота меньшей трапеции ОВСЕ — н 1 , большей АОЕД — н 2 .

Поскольку площади этих двух трапеций соотносятся как 1 к 5, то можно записать такое равенство:

(х + а 2) * н 1 = 1/5 (х + а 1) * н 2

н 1 /н 2 = (х + а 1) / (5(х + а 2)).

Высоты и стороны треугольников пропорциональны по построению. Поэтому можно записать еще одно равенство:

н 1 /н 2 = (х — а 2) / (а 1 — х).

В двух последних записях в левой части стоят равные величины, значит, можно написать, что (х + а 1) / (5(х + а 2)) равно (х — а 2) / (а 1 — х).

Здесь требуется провести ряд преобразований. Сначала перемножить крест накрест. Появятся скобки, которые укажут на разность квадратов, после применения этой формулы получится короткое уравнение.

В нем нужно раскрыть скобки и перенести все слагаемые с неизвестной «х» в левую сторону, а потом извлечь квадратный корень.

Ответ : х = √ {(а 1 2 + 5 а 2 2) / 6}.

Этот калькулятор рассчитал 2192 задачи на тему «Площадь трапеции»

ПЛОЩАДЬ ТРАПЕЦИИ

Выберете формулу вычисления площади трапеции, которую Вы планируете применить для решения поставленной перед Вами задачи:

Общая теория для вычисления площади трапеции.

Трапеция — это плоская фигура, состоящая из четырех точек, три из которых не лежат на одной прямой, и четырех отрезков (сторон), соединяющих попарно эти четыре точки, у которой две противоположные стороны параллельны (лежат на параллельных прямых), а две другие не параллельны.

Точки называются вершинами трапеции и обозначаются заглавными латинскими буквами.

Отрезки называются сторонами трапеции и обозначаются парой заглавных латинских букв соответственно вершинам, которые отрезки соединяют.

Две параллельные стороны трапеции называются основаниями трапеции .

Две не параллельные стороны трапеции называются боковыми сторонами трапеции .

Рисунок №1: Трапеция ABCD

На рисунке №1 представлена трапеция ABCD с вершинами A,B ,C, D и сторонами AB, BC, CD, DA.

AB ǁ DC — основания трапеции ABCD.

AD, BC — боковые стороны трапеции ABCD.

Угол, образованный лучами AB и AD, называется углом при вершине A. Обозначается он как ÐA или ÐBAD, или ÐDAB.

Угол, образованный лучами BA и BC, называется углом при вершине B. Обозначается он как ÐB или ÐABC, или ÐCBA.

Угол, образованный лучами CB и CD, называется углом при вершине C. Обозначается он как ÐC или ÐDCB, или ÐBCD.

Угол, образованный лучами AD и CD, называется углом при вершине D. Обозначается он как ÐD или ÐADC, или ÐCDA.

Рисунок №2: Трапеция ABCD

На рисунке №2 отрезок MN, соединяющий середины боковых сторон, называется средней линией трапеции.

Средняя линия трапеции параллельна основаниям и равна их полусумме. То есть,.


Рисунок №3: Равнобедренная трапеция ABCD

На Рисунке №3, AD=BC.

Трапеция называется равнобедренной (равнобокой) , если ее боковые стороны равны.

Рисунок №4: Прямоугольная трапеция ABCD

На Рисунке №4 угол D — прямой (равен 90 о).

Трапеция называется прямоугольной, если угол при боковой стороне прямой.

Площадью S плоской фигуры, к которым относится и трапеция, называется ограниченное замкнутое пространство на плоскости. Площадь плоской фигуры показывает величину этой фигуры.

Площадь обладает несколькими свойствами:

1. Она не может быть отрицательной.

2. Если дана некоторая замкнутая область на плоскости, которая составлена из нескольких фигур, не пересекающихся друг с другом (то есть, фигуры не имеют общих внутренних точек, но вполне могут касаться друг друга), то площадь такой области равна сумме площадей составляющих ее фигур.

3. Если две фигуры равны, то и площади их равны.

4. Площадь квадрата, который построен на единичном отрезке, равна единице.

За единицу измерения площади принимают площадь квадрата, сторона которого равна единице измерения отрезков.

При решении задач часто используются следующие формулы вычисления площади трапеции:

1. Площадь трапеции равна полусумме ее оснований умноженной на высоту:

2. Площадь трапеции равна произведению ее средней линии на высоту:

3. При известных длинах оснований и боковых сторон трапеции её площадь можно вычислить по формуле:

4. Возможно вычислить площадь равнобедренной трапеции при известной длине радиуса вписанной в трапецию окружности и известном значении угла при основании по следующей формуле:

Пример 1: Вычислить площадь трапеции с основаниями a=7, b=3 и высотой h=15.

Решение:

Ответ:

Пример 2: Найти сторону основания трапеции с площадью S=35 см 2 , высотой h=7см и вторым основанием b = 2 см.

Решение:

Для нахождения стороны основания трапеции воспользуемся формулой вычисления площади:

Выразим из данной формулы сторону основания трапеции:

Таким образом, имеем следующее:

Ответ:

Пример 3: Найти высоту трапеции с площадью S=17 см 2 и основаниями a=30 см, b = 4 см.

Решение:

Для нахождения высоты трапеции воспользуемся формулой вычисления площади:

Таким образом, имеем следующее:

Ответ:

Пример 4: Вычислить площадь трапеции с высотой h=24 и средней линией m=5.

Решение:

Для нахождения площади трапеции воспользуемся следующей формулой вычисления площади:

Таким образом, имеем следующее:

Ответ:

Пример 5: Найти высоту трапеции с площадью S = 48 см 2 и средней линией m=6 см.

Решение:

Для нахождения высоты трапеции воспользуемся формулой вычисления площади трапеции:

Выразим из данной формулы высоту трапеции:

Таким образом, имеем следующее:

Ответ:

Пример 6: Найти среднюю линию трапеции с площадью S = 56 и высотой h=4.

Решение:

Для нахождения средней линии трапеции воспользуемся формулой вычисления площади трапеции:

Выразим из данной формулы среднюю линию трапеции:

Таким образом, имеем следующее.

Формула площади трапеции через стороны. Все варианты того, как найти площадь трапеции

Что такое равнобедренная трапеция? Это геометрическая фигура, противолежащие не параллельные стороны которой равны. Существует несколько различных формул для нахождения площади трапеции с различными условиями, которые даны в задачах. То есть площадь найти можно, если дана высота, стороны, углы, диагонали и т.д. Также нельзя не упомянуть, что для равнобедренных трапеций существует некоторые “исключение”, благодаря которым поиск площади и сама формула значительно упрощается. Ниже описаны подробные решения каждого случая с примерами.

Необходимые свойства для нахождения площади равнобедренной трапеции

Мы уже выяснили, что геометрическая фигура, имеющая противолежащие не параллельные, но равные стороны – это трапеция, причем, равнобедренная. Существуют специальные случаи, когда трапеция считается равнобедренной.

  • Это условия равенства углов. Итак, обязательный пункт: углы при основании (возьмем рисунок ниже) должны быть равны. В нашем случае угол ВАD = углу CDA, a угол ABC = углу BCD
  • Второе важное правило – в подобной трапеции диагонали должны быть равны. Следовательно, АС = ВD.
  • Третий аспект: противоположные углы трапеции в сумме должны давать 180 градусов. Это значит, что угол ABC + угол CDA = 180 градусов. С углами BCD и BAD аналогично.
  • В-четвертых, если трапеция допускает описание вокруг нее окружности – то она равнобедренная.

Как найти площадь равнобедренной трапеции – формулы и их описание

  • S = (a+b)h/2 – это самая распространенная формула для нахождения площади, где а – нижнее основание, b – верхнее основание, а h – это высота.


  • Если высота неизвестна, то искать ее можно по подобной формуле: h = с*sin(x), где с это либо AB, либо CD. sin(x) – это синус угла при любом основании, то есть угол DAB = угол CDA = x. В конечном итоге формула принимает вот такой вид: S = (a+b)*с*sin(x)/2.
  • Высота также может находиться по этой формуле:

  • Итоговая формула имеет такой вид:

Рассмотрим условие, когда в трапецию будет вписана окружность.


В случае, изображенном на картинке,

QN = D = H – диаметр окружности и одновременно высота трапеции;

LO, ON, OQ = R – радиусы окружности;

DC = a – верхнее основание;

AB = b – нижнее основание;

DAB, ABC, BCD, CDA – альфа, бета – углы оснований трапеции.

Подобный случай допускает нахождение площади по таким формулам:

  • Теперь попробуем найти площадь через диагонали и углы между ними.

На рисунке обозначим AC, DB – диагонали – d. Углы COB, DOB – альфа; DOC, AOB – бета. Формула площади равнобедренной трапеции через диагонали и угол между ними, (S ) такова:

Существует множество способов найти площадь трапеции. Обычно репетитор по математике владеет несколькими приемами ее вычисления, остановимся на них подробнее:
1) , где AD и BC основания, а BH-высота трапеции. Доказательство: проведем диагональ BD и выразим площади треугольников ABD и CDB через полупроизведение их оснований на высоту:

, где DP – внешняя высота в

Сложим почленно эти равенства и учитывая, что высоты BH и DP равны, получим:

Вынесем за скобку

Что и требовалось доказать.

Следствие из формулы площади трапеции:
Так как полусумма оснований равна MN — средней линии трапеции, то

2) Применение общей формулы площади четырехугольника .
Площадь четырехугольника равна половине произведения диагоналей, умноженной на синус угла между ними
Для доказательства достаточно разбить трапецию на 4 треугольника, выразить площадь каждого через «половину произведения диагоналей на синус угла между ними» (в качестве угла берется , сложить получившиеся выражения, вынести за скобку и раскладываю эту скобку на множители методом группировки получить ее равенство выражению . Отсюда

3) Метод сдвига диагонали
Это мое название. В школьных учебниках репетитор по математике не встретит такого заголовка. Описание приема можно найти только в дополнительных учебных пособиях в качестве примера решения какой-нибудь задачи. Отмечу, что большинство интересных и полезных фактов планиметрии репетиторы по математике открывают ученикам в процессе выполнения практической работы. Это крайне неоптимально, ибо школьнику нужно выделять их в отдельные теоремы и называть «громкими именами». Одно из таких – «сдвиг диагонали». О чем идет речь? Проведем через вершину B прямую параллельную к АС до пересечения с нижним основанием в точке E. В таком случае четырехугольник EBCA будет параллелограммом (по определению) и поэтому BC=EA и EB=AC. Нам сейчас важно первое равенство. Имеем:

Заметим, что треугольник BED, площадь которого равна площади трапеции, имеет еще несколько замечательных свойств:
1) Его площадь равна площади трапеции
2) Его равнобедренность происходит одновременно с равнобедренность самой трапеции
3) Верхний его угол при вершине B равен углу между диагоналями трапеции (что очень часто используется в задачах)
4) Его медиана BK равна расстоянию QS между серединами оснований трапеции. С применением этого свойства я недавно столкнулся при подготовке ученика на мехмат МГУ по учебнику Ткачука, вариант 1973 года (задача приводится внизу страницы).

Спецприемы репетитора по математике.

Иногда я предлагаю задачи на весьма хитрый путь нахождении я площади трапеции. Я отношу его к спецприемам ибо на практике репетитор их использует крайне редко. Если вам нужна подготовка к ЕГЭ по математике только в части B, можно про них и не читать. Для остальных рассказываю дальше. Оказывается площадь трапеции в два раза больше площади треугольника с вершинами в концах одной боковой стороны и серединой другой, то есть треугольника ABS на рисунке:
Доказательство: проведем высоты SM и SN в треугольниках BCS и ADS и выразим сумму площадей этих треугольников:

Так как точка S – середина CD, то (докажите это сами).Найдем cумму площадей треугольников:

Так как эта сумма оказалась равной половине площади трапеции, то — вторая ее половина. Ч.т.д.

В копилку спецприемов репетитора я бы отнес форму вычисления площади равнобедренной трапеции по ее сторонам: где p – полупериметр трапеции. Доказательство я приводить не буду. Иначе ваш репетитор по математике останется без работы:). Приходите на занятия!

Задачи на площадь трапеции:

Замечание репетитора по математике : Нижеприведенный список не является методическим сопровождением к теме, это только небольшая подборка интересных задач на вышерассмотренные приемы.

1) Нижнее основание равнобедренной трапеции равно 13, а верхнее равно 5. Найдите площадь трапеции, если ее диагональ перпендикулярна боковой стороне.
2) Найдите площадь трапеции, если ее основания равны 2см и 5см, а боковые стороны 2см и 3см.
3) В равнобокой трапеции большее основание равно 11, боковая сторона равна 5, а диагональ равна Найти площадь трапеции.
4) Диагональ равнобокой трапеции равна 5, а средняя линия равна 4. Найти площадь.
5) В равнобедренной трапеции основания равны 12 и 20, а диагонали взаимно перпендикулярны. Вычислить площадь трапеции
6) Диагональ равнобокой трапеции составляет с ее нижним основанием угол . Найти площадь трапеции, если ее высота равна 6см.
7) Площадь трапеции равна 20, а одна из ее боковых сторон равна 4 см. Найдите расстояние до нее от середины противоположной боковой стороны.
8) Диагональ равнобокой трапеции делит ее на треугольники с площадями 6 и 14. Найти высоту, если боковая сторона равна 4.
9) В трапеции диагонали равны 3 и 5, а отрезок, соединяющий середины оснований равен 2. Найти площадь трапеции (Мехмат МГУ, 1970г).

Я выбирал не самые сложные задачи (не стоит пугаться мехмата!) с расчетом на возможность их самостоятельного решения. Решайте на здоровье! Если вам нужна подготовка к ЕГЭ по математике, то без участия в этом процессе формулы площади трапеции могут возникнуть серьезные проблемы даже с задачей B6 и тем более с C4. Не запускайте тему и в случае каких-либо затруднений обращайтесь за помощью. Репетитор по математике всегда рад вам помочь.

Колпаков А.Н.
Репетитор по математике в Москве , подготовка к ЕГЭ в Строгино .

Трапецией называется четырехугольник, у которого только две стороны параллельны между собой.

Они называются основаниями фигуры, оставшиеся – боковыми сторонами. Частными случаями фигуры считается параллелограмм. Также существует криволинейная трапеция, которая включает в себя график функции. Формулы площади трапеции включают в себя практически все ее элементы, и лучшее решение подбирается в зависимости от заданных величин.
Основные роли в трапеции отводятся высоте и средней линии. Средняя линия – это линия, соединяющая середины боковых сторон. Высота трапеции проводится под прямым углом от верхнего угла к основанию.
Площадь трапеции через высоту равняется произведению полусуммы длин оснований, умноженному на высоту:

Если по условиям известна средняя линия, то эта формула значительно упрощается, так как она равна полусумме длин оснований :

Если по условиям даны длины всех сторон, то можно рассмотреть пример расчета площади трапеции через эти данные:

Допустим, дана трапеция с основаниями a = 3 см, b = 7 см и боковыми сторонами c = 5 см, d = 4 см. найдем площадь фигуры:

Площадь равнобокой трапеции


Отдельным случаем считается равнобокая или, как ее еще называют, равнобедренная трапеция.
Особым случаем является и нахождение площади равнобедренной (равнобокой) трапеции. Формула выводится различными способами – через диагонали, через углы, прилегающие к основанию и радиус вписанной окружности.
Если по условиям задана длина диагоналей и известен угол между ними можно использовать такую формулу:

Помните, что диагонали равнобокой трапеции равны между собой!


То есть, зная одно их оснований, сторону и угол, можно легко рассчитать площадь.

Площадь криволинейной трапеции


Отдельный случай – это криволинейная трапеция . Она располагается на оси координат и ограничивается графиком непрерывной положительной функции.

Ее основание располагает на оси X и ограничивается двумя точками:
Интегралы помогают вычислить площадь криволинейной трапеции.
Формула прописывается так:

Рассмотрим пример расчета площади криволинейной трапеции. Формула требует определенных знаний для работы с определенными интегралами. Для начала разберем значение определенного интеграла:

Здесь F(a) – это значение первообразной функции f(x) в точке a , F(b) – значение этой же функции f(x) в точке b .

Теперь решим задачу. На рисунке изображена криволинейная трапеция, ограниченная функцией . Функция
Нам необходимо найти площадь выделенной фигуры, которая является криволинейной трапецией, ограниченной сверху графиком , справа прямой x ={-8}, слева прямой x ={-10} и осью OX снизу.
Площадь этой фигуры мы будем рассчитывать по формуле:

Условиями задачи нам задана функция. По ней мы найдем значения первообразной в каждой из наших точек:

Теперь
Ответ: площадь заданной криволинейной трапеции равняется 4.

Ничего сложного в расчетах этого значения нет. Важна только предельная внимательность в вычислениях.

В математике известно несколько видов четырехугольников: квадрат, прямоугольник, ромб, параллелограмм. Среди них и трапеция — вид выпуклого четырехугольника, у которого две стороны параллельны, а две другие нет. Параллельные противоположные стороны называются основаниями, а две другие – боковыми сторонами трапеции. Отрезок, который соединяет середины боковых сторон, называется средней линией. Существует несколько видов трапеций: равнобедренная, прямоугольная, криволинейная. Для каждого вида трапеции есть формулы для нахождения площади.

Площадь трапеции

Чтобы найти площадь трапеции, нужно знать длину ее оснований и высоту. Высота трапеции — это отрезок, перпендикулярный основаниям. Пусть верхнее основание — a, нижнее основание — b, а высота — h. Тогда вычислить площадь S можно по формуле:

S = ½ * (a+b) * h

т.е. взять полусумму оснований, умноженную на высоту.

Также удастся вычислить площадь трапеции, если известно значение высоты и средней линии. Обозначим среднюю линию — m. Тогда

Решим задачу посложнее: известны длины четырех сторон трапеции — a, b, c, d. Тогда площадь отыщется по формуле:


Если известны длины диагоналей и угол между ними, то площадь ищется так:

S = ½ * d1 * d2 * sin α

где d с индексами 1 и 2 — диагонали. В данной формуле в расчете приводится синус угла.

При известных длинах оснований a и b и двух углах при нижнем основании площадь вычисляется так:

S = ½ * (b2 — a2) * (sin α * sin β / sin(α + β))

Площадь равнобедренной трапеции

Равнобедренная трапеция — это частный случай трапеции. Ее отличие в том, что такая трапеция — это выпуклый четырехугольник с осью симметрии, проходящей через середины двух противоположных сторон. Ее боковые стороны равны.


Найти площадь равнобедренной трапеции можно несколькими способами.

  • Через длины трех сторон. В этом случае длины боковых сторон будут совпадать, поэтому обозначены одной величиной — с, а и b — длины оснований:

  • Если известна длина верхнего основания, боковой стороны и величина угла при нижнем основании, то площадь вычисляется так:

S = c * sin α * (a + c * cos α)

где а — верхнее основание, с — боковая сторона.

  • Если вместо верхнего основания известна длина нижнего – b, площадь рассчитывается по формуле:

S = c * sin α * (b – c * cos α)

  • Если когда известны два основания и угол при нижнем основании, площадь вычисляется через тангенс угла:

S = ½ * (b2 – a2) * tg α

  • Также площадь рассчитывается через диагонали и угол между ними. В этом случае диагонали по длине равны, поэтому каждую обозначаем буквой d без индексов:

S = ½ * d2 * sin α

  • Вычислим площадь трапеции, зная длину боковой стороны, средней линии и величину угла при нижнем основании.

Пусть боковая сторона — с, средняя линия — m, угол — a, тогда:

S = m * c * sin α

Иногда в равностороннюю трапецию можно вписать окружность, радиус которой будет — r.


Известно, что в любую трапецию можно вписать окружность, если сумма длин оснований равна сумме длин ее боковых сторон. Тогда площадь найдется через радиус вписанной окружности и угол при нижнем основании:

S = 4r2 / sin α

Такой же расчет производится и через диаметр D вписанной окружности (кстати, он совпадает с высотой трапеции):

Зная основания и угол, площадь равнобедренной трапеции вычисляется так:

S = a * b / sin α

(эта и последующие формулы верны только для трапеций с вписанной окружностью).


Через основания и радиус окружности площадь ищется так:

Если известны только основания, то площадь считается по формуле:


Через основания и боковую линию площадь трапеции с вписанным кругом и через основания и среднюю линию — m вычисляется так:

Площадь прямоугольной трапеции

Прямоугольной называется трапеция, у которой одна из боковых сторон перпендикулярна основаниям. В этом случае боковая сторона по длине совпадает с высотой трапеции.

Прямоугольная трапеция представляет из себя квадрат и треугольник. Найдя площадь каждой из фигур, сложите полученные результаты и получите общую площадь фигуры.


Также для вычисления площади прямоугольной трапеции подходят общие формулы для расчета площади трапеции.

  • Если известны длины оснований и высота (или перпендикулярная боковая сторона), то площадь рассчитывается по формуле:

S = (a + b) * h / 2

В качестве h (высоты) может выступать боковая сторона с. Тогда формула выглядит так:

S = (a + b) * c / 2

  • Другой способ рассчитать площадь — перемножить длину средней линии на высоту:

или на длину боковой перпендикулярной стороны:

  • Следующий способ вычисления — через половину произведения диагоналей и синус угла между ними:

S = ½ * d1 * d2 * sin α


Если диагонали перпендикулярны, то формула упрощается до:

S = ½ * d1 * d2

  • Еще один способ вычисления — через полупериметр (сумма длин двух противоположных сторон) и радиус вписанной окружности.

Эта формула действительна для оснований. Если брать длины боковых сторон, то одна из них будет равна удвоенному радиусу. Формула будет выглядеть так:

S = (2r + c) * r

  • Если в трапецию вписана окружность, то площадь вычисляется так же:

где m — длина средней линии.

Площадь криволинейной трапеции

Криволинейная трапеция представляет из себя плоскую фигуру, ограниченную графиком неотрицательной непрерывной функции y = f(x), определенной на отрезке , осью абсцисс и прямыми x = a, x = b. По сути, две ее стороны параллельны друг другу (основания), третья сторона перпендикулярна основаниям, а четвертая представляет из себя кривую, соответствующую графику функции.


Площадь криволинейной трапеции ищут через интеграл по формуле Ньютона-Лейбница:


Так вычисляются площади различных видов трапеций. Но, помимо свойств сторон, трапеции обладают одинаковыми свойствами углов. Как у всех существующих четырехугольников, сумма внутренних углов трапеции равна 360 градусов. А сумма углов, прилежащих к боковой стороне, — 180 градусам.

Площадь трапеции если известна средняя линия. Площадь трапеции

Многоликая трапеция… Она может быть произвольной, равнобедренной или прямоугольной. И в каждом случае нужно знать, как найти площадь трапеции. Конечно, проще всего запомнить основные формулы. Но иногда проще воспользоваться той, которая выведена с учетом всех особенностей конкретной геометрической фигуры.

Несколько слов о трапеции и ее элементах

Любой четырехугольник, у которого две стороны параллельны, можно назвать трапецией. В общем случае они не равны и называются основаниями. Большее из них — нижнее, а другое — верхнее.

Две другие стороны оказываются боковыми. У произвольной трапеции они имеют различную длину. Если же они равны, то фигура становится равнобедренной.

Если вдруг угол между любой боковой стороной и основанием окажется равным 90 градусам, то трапеция является прямоугольной.

Все эти особенности могут помочь в решении задачи о том, как найти площадь трапеции.

Среди элементов фигуры, которые могут оказаться незаменимыми в решении задач, можно выделить такие:

  • высота, то есть отрезок, перпендикулярный обоим основаниям;
  • средняя линия, которая имеет своими концами середины боковых сторон.

По какой формуле вычислить площадь, если известны основания и высота?

Это выражение дается основным, потому что чаще всего можно узнать эти величины, даже когда они не даны явно. Итак, чтобы понять, как найти площадь трапеции, потребуется сложить оба основания и разделить их на два. Получившееся значение потом еще умножить на значение высоты.

Если обозначить основания буквами а 1 и а 2 , высоту — н, то формула для площади будет выглядеть так:

S = ((а 1 + а 2)/2)*н.

Формула, по которой вычисляется площадь, если даны ее высота и средняя линия

Если посмотреть внимательно на предыдущую формулу, то легко заметить, что в ней явно присутствует значение средней линии. А именно, сумма оснований, деленная на два. Пусть средняя линия будет обозначена буквой l, тогда формула для площади станет такой:

S = l * н.

Возможность найти площадь по диагоналям

Этот способ поможет, если известен угол, образованный ими. Предположим, что диагонали обозначены буквами д 1 и д 2 , а углы между ними — α и β. Тогда формула того, как найти площадь трапеции, будет записана следующим образом:

S = ((д 1 * д 2)/2) * sin α.

В этом выражении можно легко заменить α на β. Результат не изменится.

Как узнать площадь, если известны все стороны фигуры?

Бывают и такие ситуации, когда в этой фигуре известны именно стороны. Эта формула получается громоздкой и ее сложно запомнить. Но возможно. Пусть боковые стороны имеют обозначение: в 1 и в 2 , основание а 1 больше, чем а 2 . Тогда формула площади примет такой вид:

S = ((а 1 + а 2) / 2) * √ {в 1 2 — [(а 1 — а 2) 2 + в 1 2 — в 2 2) / (2 * (а 1 — а 2))] 2 }.

Способы вычисления площади равнобедренной трапеции

Первый связан с тем, что в нее можно вписать окружность. И, зная ее радиус (он обозначается буквой r), а также угол при основании — γ, можно воспользоваться такой формулой:

S = (4 * r 2) / sin γ.

Последняя общая формула, которая основана на знании всех сторон фигуры, существенно упростится за счет того, что боковые стороны имеют одинаковое значение:

S = ((а 1 + а 2) / 2) * √ {в 2 — [(а 1 — а 2) 2 / (2 * (а 1 — а 2))] 2 }.

Методы вычисления площади прямоугольной трапеции

Понятно, что подойдет любой из перечисленных для произвольной фигуры. Но иногда полезно знать об одной особенности такой трапеции. Она заключается в том, что разность квадратов длин диагоналей равна разности, составленной из квадратов оснований.

Часто формулы для трапеции забываются, в то время как выражения для площадей прямоугольника и треугольника помнятся. Тогда можно применить простой способ. Разделить трапецию на две фигуры, если она прямоугольная, или три. Одна точно будет прямоугольником, а вторая, или две оставшиеся, треугольниками. После вычисления площадей этих фигур останется их только сложить.

Это достаточно простой способ того, как найти площадь прямоугольной трапеции.

Как быть, если известны координаты вершин трапеции?

В этом случае потребуется воспользоваться выражением, которое позволяет определить расстояние между точками. Его можно применить три раза: для того, чтобы узнать оба основания и одну высоту. А потом просто применить первую формулу, которая описана немного выше.

Для иллюстрации такого метода можно привести такой пример. Даны вершины с координатами А(5; 7), В(8; 7), С(10; 1), Д(1; 1). Нужно узнать площадь фигуры.

До того как найти площадь трапеции, по координатам нужно вычислить длины оснований. Потребуется такая формула:

длина отрезка = √{(разность первых координат точек) 2 + (разность вторых координат точек) 2 }.

Верхнее основание обозначено АВ, значит, его длина будет равна √{(8-5) 2 + (7-7) 2 } = √9 = 3. Нижнее — СД = √ {(10-1) 2 + (1-1) 2 } = √81 = 9.

Теперь нужно провести высоту из вершины на основание. Пусть ее начало будет в точке А. Конец отрезка окажется на нижнем основании в точке с координатами (5; 1), пусть это будет точка Н. Длина отрезка АН получится равной √{(5-5) 2 + (7-1) 2 } = √36 = 6.

Осталось только подставить получавшиеся значения в формулу площади трапеции:

S = ((3 + 9) / 2) * 6 = 36.

Задача решена без единиц измерения, потому что не указан масштаб координатной сетки. Он может быть как миллиметр, так и метр.

Примеры задач

№ 1. Условие. Известен угол между диагоналями произвольной трапеции, он равен 30 градусам. Меньшая диагональ имеет значение 3 дм, а вторая больше ее в 2 раза. Необходимо посчитать площадь трапеции.

Решение. Для начала нужно узнать длину второй диагонали, потому что без этого не удастся сосчитать ответ. Вычислить ее несложно, 3 * 2 = 6 (дм).

Теперь нужно воспользоваться подходящей формулой для площади:

S = ((3 * 6) / 2) * sin 30º = 18/2 * ½ = 4,5 (дм 2). Задача решена.

Ответ: площадь трапеции равна 4,5 дм 2 .

№ 2. Условие. В трапеции АВСД основаниями являются отрезки АД и ВС. Точка Е — середина стороны СД. Из нее проведен перпендикуляр к прямой АВ, конец этого отрезка обозначен буквой Н. Известно, что длины АВ и ЕН равны соответственно 5 и 4 см. Нужно вычислить площадь трапеции.

Решение. Для начала нужно сделать чертеж. Поскольку значение перпендикуляра меньше стороны, к которой он проведен, то трапеция будет немного вытянутой вверх. Так ЕН окажется внутри фигуры.

Чтобы отчетливо увидеть ход решения задачи, потребуется выполнить дополнительное построение. А именно, провести прямую, которая будет параллельна стороне АВ. Точки пересечения этой прямой с АД — Р, а с продолжением ВС — Х. Получившаяся фигура ВХРА — параллелограмм. Причем его площадь равна искомой. Это связано с тем, что треугольники, которые получились при дополнительном построении, равны. Это следует из равенства стороны и двух прилежащих к ней углов, один — вертикальный, другой — накрест лежащий.

Найти площадь параллелограмма можно по формуле, которая содержит произведение стороны и высоты, опущенной на нее.

Таким образом, площадь трапеции равна 5 * 4 = 20 см 2 .

Ответ: S = 20 см 2 .

№ 3. Условие. Элементы равнобедренной трапеции имеют такие значения: нижнее основание — 14 см, верхнее — 4 см, острый угол — 45º. Нужно вычислить ее площадь.

Решение. Пусть меньшее основание имеет обозначение ВС. Высота, проведенная из точки В, будет называться ВН. Поскольку угол 45º, то треугольник АВН получится прямоугольный и равнобедренный. Значит, АН=ВН. Причем АН очень легко найти. Она равна половине разности оснований. То есть (14 — 4) / 2 = 10 / 2 = 5 (см).

Основания известны, высота сосчитана. Можно пользоваться первой формулой, которая здесь была рассмотрена для произвольной трапеции.

S = ((14 + 4) / 2) * 5 = 18/2 * 5 = 9 * 5 = 45 (см 2).

Ответ: Искомая площадь равна 45 см 2 .

№ 4. Условие. Имеется произвольная трапеция АВСД. На ее боковых сторонах взяты точки О и Е, так что ОЕ параллельна основанию АД. Площадь трапеции АОЕД в пять раз больше, чем у ОВСЕ. Вычислить значение ОЕ, если известны длины оснований.

Решение. Потребуется провести две параллельные АВ прямые: первую через точку С, ее пересечение с ОЕ — точка Т; вторую через Е и точкой пересечения с АД будет М.

Пусть неизвестная ОЕ=х. Высота меньшей трапеции ОВСЕ — н 1 , большей АОЕД — н 2 .

Поскольку площади этих двух трапеций соотносятся как 1 к 5, то можно записать такое равенство:

(х + а 2) * н 1 = 1/5 (х + а 1) * н 2

н 1 /н 2 = (х + а 1) / (5(х + а 2)).

Высоты и стороны треугольников пропорциональны по построению. Поэтому можно записать еще одно равенство:

н 1 /н 2 = (х — а 2) / (а 1 — х).

В двух последних записях в левой части стоят равные величины, значит, можно написать, что (х + а 1) / (5(х + а 2)) равно (х — а 2) / (а 1 — х).

Здесь требуется провести ряд преобразований. Сначала перемножить крест накрест. Появятся скобки, которые укажут на разность квадратов, после применения этой формулы получится короткое уравнение.

В нем нужно раскрыть скобки и перенести все слагаемые с неизвестной «х» в левую сторону, а потом извлечь квадратный корень.

Ответ : х = √ {(а 1 2 + 5 а 2 2) / 6}.

И . Теперь можно приступить к рассмотрению вопроса как найти площадь трапеции. Данная задача в быту возникает очень редко, но иногда оказывается необходимой, к примеру, чтобы найти площадь комнаты в форме трапеции, которые все чаще применяют при строительстве современных квартир, или в дизайн-проектах по ремонту.

Трапеция — это геометрическая фигура, образованная четырьмя пересекающимися отрезками, два из которых параллельны между собой и называются основаниями трапеции. Два других отрезка называются сторонами трапеции. Кроме того, в дальнейшем нам пригодится еще одно определение. Это средняя линия трапеции, которая представляет собой отрезок, соединяющий середины боковых сторон и высота трапеции, которая равна расстоянию между основаниями.
Как и у треугольников, у трапеция есть частные виды в виде равнобедренной (равнобокой) трапеции, у которой длина боковых сторон одинаковы и прямоугольной трапеции, у которой одна из сторон образует с основаниями прямой угол.

Трапеции обладают некоторыми интересными свойствами:

  1. Средняя линия трапеции равна полусумме оснований и параллельна им.
  2. У равнобедренных трапеций боковые стороны и углы которые они образуют с основаниями равны.
  3. Середины диагоналей трапеции и точка пересечения ее диагоналей находятся на одной прямой.
  4. Если сумма боковых сторон трапеции равна сумме оснований, то в нее можно вписать круг
  5. Если сумма углов, образованных сторонами трапеции у любого ее основания равна 90, то длина отрезка, соединяющего середины оснований, равна их полуразности.
  6. Равнобедренную трапецию можно описать окружностью. И наоборот. Если в трапеция вписывается в окружность, значит она равнобедренная.
  7. Отрезок, проходящий через середины оснований равнобедренной трапеции будет перпендикулярен ее основаниям и представляет собой ось симетрии.

Как найти площадь трапеции .

Площадь трапеции будет равна полусумме ее оснований, умноженной на высоту. В виде формулы это записывается в виде выражения:

где S-площадь трапеции, a,b-длина каждого из оснований трапеции, h-высота трапеции.


Понять и запомнить эту формулу можно следующим образом. Как следует из рисунка ниже трапецию с использованием средней линии можно преобразовать в прямоугольник, длина которого и будет равна полусумме оснований.

Можно также любую трапецию разложить на более простые фигуры: прямоугольник и один, или два треугольника и если вам так проще, то найти площадь трапеции, как сумму площадей составляющих ее фигур.

Есть еще одна простая формула для подсчета ее площади. Согласно ней площадь трапеции равна произведению ее средней линии на высоту трапеции и записывается в виде: S = m*h, где S-площадь, m-длина средней линии, h-высота трапеции. Данная формула больше подходит для задач по математике, чем для бытовых задач, так как в реальных условиях вам не будет известна длина средней линии без предварительных расчетов. А известны вам будут только длины оснований и боковых сторон.

В этом случае площадь трапеции может быть найдена по формуле:

S = ((a+b)/2)*√c 2 -((b-a) 2 +c 2 -d 2 /2(b-a)) 2

где S-площадь, a,b-основания, c,d-боковые стороны трапеции.

Существуют еще несколько способов того, как найти площади трапеции. Но, они примерно также неудобны как и последняя формула, а значит не имеет смысла на них останавливаться. Поэтому, рекомендуем вам пользоваться первой формулой из статьи и желаем всегда получать точные результаты.

Существует множество способов найти площадь трапеции. Обычно репетитор по математике владеет несколькими приемами ее вычисления, остановимся на них подробнее:
1) , где AD и BC основания, а BH-высота трапеции. Доказательство: проведем диагональ BD и выразим площади треугольников ABD и CDB через полупроизведение их оснований на высоту:

, где DP – внешняя высота в

Сложим почленно эти равенства и учитывая, что высоты BH и DP равны, получим:

Вынесем за скобку

Что и требовалось доказать.

Следствие из формулы площади трапеции:
Так как полусумма оснований равна MN — средней линии трапеции, то

2) Применение общей формулы площади четырехугольника .
Площадь четырехугольника равна половине произведения диагоналей, умноженной на синус угла между ними
Для доказательства достаточно разбить трапецию на 4 треугольника, выразить площадь каждого через «половину произведения диагоналей на синус угла между ними» (в качестве угла берется , сложить получившиеся выражения, вынести за скобку и раскладываю эту скобку на множители методом группировки получить ее равенство выражению . Отсюда

3) Метод сдвига диагонали
Это мое название. В школьных учебниках репетитор по математике не встретит такого заголовка. Описание приема можно найти только в дополнительных учебных пособиях в качестве примера решения какой-нибудь задачи. Отмечу, что большинство интересных и полезных фактов планиметрии репетиторы по математике открывают ученикам в процессе выполнения практической работы. Это крайне неоптимально, ибо школьнику нужно выделять их в отдельные теоремы и называть «громкими именами». Одно из таких – «сдвиг диагонали». О чем идет речь? Проведем через вершину B прямую параллельную к АС до пересечения с нижним основанием в точке E. В таком случае четырехугольник EBCA будет параллелограммом (по определению) и поэтому BC=EA и EB=AC. Нам сейчас важно первое равенство. Имеем:

Заметим, что треугольник BED, площадь которого равна площади трапеции, имеет еще несколько замечательных свойств:
1) Его площадь равна площади трапеции
2) Его равнобедренность происходит одновременно с равнобедренность самой трапеции
3) Верхний его угол при вершине B равен углу между диагоналями трапеции (что очень часто используется в задачах)
4) Его медиана BK равна расстоянию QS между серединами оснований трапеции. С применением этого свойства я недавно столкнулся при подготовке ученика на мехмат МГУ по учебнику Ткачука, вариант 1973 года (задача приводится внизу страницы).

Спецприемы репетитора по математике.

Иногда я предлагаю задачи на весьма хитрый путь нахождении я площади трапеции. Я отношу его к спецприемам ибо на практике репетитор их использует крайне редко. Если вам нужна подготовка к ЕГЭ по математике только в части B, можно про них и не читать. Для остальных рассказываю дальше. Оказывается площадь трапеции в два раза больше площади треугольника с вершинами в концах одной боковой стороны и серединой другой, то есть треугольника ABS на рисунке:
Доказательство: проведем высоты SM и SN в треугольниках BCS и ADS и выразим сумму площадей этих треугольников:

Так как точка S – середина CD, то (докажите это сами).Найдем cумму площадей треугольников:

Так как эта сумма оказалась равной половине площади трапеции, то — вторая ее половина. Ч.т.д.

В копилку спецприемов репетитора я бы отнес форму вычисления площади равнобедренной трапеции по ее сторонам: где p – полупериметр трапеции. Доказательство я приводить не буду. Иначе ваш репетитор по математике останется без работы:). Приходите на занятия!

Задачи на площадь трапеции:

Замечание репетитора по математике : Нижеприведенный список не является методическим сопровождением к теме, это только небольшая подборка интересных задач на вышерассмотренные приемы.

1) Нижнее основание равнобедренной трапеции равно 13, а верхнее равно 5. Найдите площадь трапеции, если ее диагональ перпендикулярна боковой стороне.
2) Найдите площадь трапеции, если ее основания равны 2см и 5см, а боковые стороны 2см и 3см.
3) В равнобокой трапеции большее основание равно 11, боковая сторона равна 5, а диагональ равна Найти площадь трапеции.
4) Диагональ равнобокой трапеции равна 5, а средняя линия равна 4. Найти площадь.
5) В равнобедренной трапеции основания равны 12 и 20, а диагонали взаимно перпендикулярны. Вычислить площадь трапеции
6) Диагональ равнобокой трапеции составляет с ее нижним основанием угол . Найти площадь трапеции, если ее высота равна 6см.
7) Площадь трапеции равна 20, а одна из ее боковых сторон равна 4 см. Найдите расстояние до нее от середины противоположной боковой стороны.
8) Диагональ равнобокой трапеции делит ее на треугольники с площадями 6 и 14. Найти высоту, если боковая сторона равна 4.
9) В трапеции диагонали равны 3 и 5, а отрезок, соединяющий середины оснований равен 2. Найти площадь трапеции (Мехмат МГУ, 1970г).

Я выбирал не самые сложные задачи (не стоит пугаться мехмата!) с расчетом на возможность их самостоятельного решения. Решайте на здоровье! Если вам нужна подготовка к ЕГЭ по математике, то без участия в этом процессе формулы площади трапеции могут возникнуть серьезные проблемы даже с задачей B6 и тем более с C4. Не запускайте тему и в случае каких-либо затруднений обращайтесь за помощью. Репетитор по математике всегда рад вам помочь.

Колпаков А.Н.
Репетитор по математике в Москве , подготовка к ЕГЭ в Строгино .

Практика прошлогодних ЕГЭ и ГИА показывает, что задачи по геометрии вызывают сложности у многих школьников. Вы легко справитесь с ними, если заучите все нужные формулы и попрактикуетесь в решении задач.

В этой статье вы увидите формулы нахождения площади трапеции, а также примеры задач с решениями. Такие же могут попасться вам в КИМах на аттестационных экзаменах или на олимпиадах. Поэтому отнеситесь к ним внимательно.

Что нужно знать про трапецию?

Для начала вспомним, что трапецией называется четырехугольник, у которого две противоположные стороны, их еще называют основаниями, параллельны, а две другие – нет.

В трапеции также может быть опущена высота (перпендикуляр к основанию). Проведена средняя линия – это прямая, которая параллельна основаниям и равна половине их суммы. А также диагонали, которые могут пересекаться, образуя острые и тупые углы. Или, в отдельных случаях, под прямым углом. Кроме того, если трапеция равнобедренная, в нее можно вписать окружность. И описать окружность около нее.

Формулы площади трапеции

Для начала рассмотрим стандартные формулы нахождения площади трапеции. Способы вычислить площадь равнобедренной и криволинейной трапеций рассмотрим ниже.

Итак, представьте, что у вас есть трапеция с основаниями a и b, в которой к большему основанию опущена высота h. Вычислить площадь фигуры в таком случае проще простого. Надо всего лишь разделить на два сумму длин оснований и умножить то, что получится, на высоту: S = 1/2(a + b)*h .

Возьмем другой случай: предположим, в трапеции, кроме высоты, проведена средняя линия m. Нам известна формула нахождения длины средней линии: m = 1/2(a + b). Поэтому с полным правом можем упростить формулу площади трапеции до следующего вида: S = m* h . Другими словами, чтобы найти площадь трапеции, надо умножить среднюю линию на высоту.

Рассмотрим еще один вариант: в трапеции проведены диагонали d 1 и d 2 , которые пересекаются не под прямым углом α. Чтобы вычислить площадь такой трапеции, вам нужно разделить на два произведение диагоналей и умножить то, что получится, на sin угла между ними: S= 1/2d 1 d 2 *sinα .

Теперь рассмотрим формулу для нахождения площади трапеции, если о ней неизвестно ничего, кроме длин всех ее сторон: a, b, c и d. Это громоздкая и сложная формула, но вам будет полезно запомнить на всякий случай и ее: S = 1/2(a + b) * √c 2 – ((1/2(b – a)) * ((b – a) 2 + c 2 – d 2)) 2 .

Кстати, приведенные выше примеры верны и для того случая, когда вам потребуется формула площади прямоугольной трапеции. Эта трапеция, боковая сторона которой примыкает к основаниям под прямым углом.

Равнобедренная трапеция

Трапеция, боковые стороны которой равны, называется равнобедренной. Мы рассмотрим несколько вариантов формулы площади равнобедренной трапеции.

Первый вариант: для случая, когда внутрь равнобедренной трапеции вписана окружность с радиусом r, а боковая сторона и большее основание образуют острый угол α. Окружность может быть вписана в трапецию при условии, что сумма длин ее оснований равна сумме длин боковых сторон.

Площадь равнобедренной трапеции вычисляется так: умножьте квадрат радиуса вписанной окружности на четыре и разделите все это на sinα: S = 4r 2 /sinα . Еще одна формула площади является частным случаем для того варианта, когда угол между большим основанием и боковой стороной равен 30 0: S = 8r 2 .

Второй вариант: на этот раз возьмем равнобедренную трапецию, в которой вдобавок проведены диагонали d 1 и d 2 , а также высота h. Если диагонали трапеции взаимно перпендикулярны, высота составляет половину суммы оснований: h = 1/2(a + b). Зная это, легко преобразовать уже знакомую вам формулу площади трапеции в такой вид: S = h 2 .

Формула площади криволинейной трапеции

Начнем с того, что разберемся: что такое криволинейная трапеция. Представьте себе ось координат и график непрерывной и неотрицательной функции f, которая не меняет знака в пределах заданного отрезка на оси x. Криволинейную трапецию образуют график функции у = f(x) – вверху, ось х – внизу (отрезок ), а по бокам – прямые, проведенные между точками a и b и графиком функции.

Вычислить площадь такой нестандартной фигуры нельзя приведенными выше способами. Тут нужно применить математический анализ и использовать интеграл. А именно: формулу Ньютона-Лейбница – S = ∫ b a f(x)dx = F(x)│ b a = F(b) – F(a) . В этой формуле F – первообразная нашей функции на выбранном отрезке . И площадь криволинейной трапеции соответствует приращению первообразной на заданном отрезке.

Примеры задач

Чтобы все эти формулы лучше улеглись в голове, вот вам несколько примеров задач на нахождение площади трапеции. Лучше всего будет, если вы сперва попробуете решить задачи сами, и только потом сверите полученный ответ с готовым решением.

Задача №1: Дана трапеция. Ее большее основание – 11 см, меньшее – 4см. В трапеции проведены диагонали, одна длиной 12 см, вторая – 9 см.

Решение: Постройте трапецию АМРС. Проведите прямую РХ через вершину Р так, чтобы она оказалась параллельной диагонали МС и пересекла прямую АС в точке Х. Получится треугольник АРХ.

Мы рассмотрим две полученных в результате этих манипуляций фигуры: треугольник АРХ и параллелограмм СМРХ.

Благодаря параллелограмму мы узнаем, что РХ = МС = 12 см и СХ = МР = 4см. Откуда можем вычислить сторону АХ треугольника АРХ: АХ = АС + СХ = 11 + 4 = 15 см.

Мы также можем доказать, что треугольник АРХ – прямоугольный (для этого примените теорему Пифагора – АХ 2 = АР 2 + РХ 2). И высчитать его площадь: S APX = 1/2(AP * PX) = 1/2(9 * 12) = 54 см 2 .

Дальше вам потребуется доказать, что треугольники АМР и РСХ являются равновеликими. Основанием послужит равенство сторон МР и СХ (уже доказанное выше). А также высоты, которые вы опустите на эти стороны – они равны высоте трапеции АМРС.

Все это позволит вам утверждать, что S AMPC = S APX = 54 см 2 .

Задача №2: Дана трапеция КРМС. На ее боковых сторонах расположены точки О и Е, при этом ОЕ и КС параллельны. Также известно, что площади трапеций ОРМЕ и ОКСЕ находятся в соотношении 1:5. РМ = а и КС = b. Требуется найти ОЕ.

Решение: Проведите через точку М прямую, параллельную РК, и точку ее пересечения с ОЕ обозначьте Т. А – точка пересечения прямой, проведенной через точку Е параллельно РК, с основанием КС.

Введем еще одно обозначение – ОЕ = х. А также высоту h 1 для треугольника ТМЕ и высоту h 2 для треугольника АЕС (вы можете самостоятельно доказать подобие этих треугольников).

Будем считать, что b > а. Площади трапеций ОРМЕ и ОКСЕ относятся как 1:5, что дает нам право составить такое уравнение: (х + а) * h 1 = 1/5(b + х) * h 2 . Преобразуем и получим: h 1 / h 2 = 1/5 * ((b + х)/(х + а)).

Раз треугольники ТМЕ и АЕС подобные, имеем h 1 / h 2 = (х – а)/(b – х). Объединим обе записи и получим: (х – а)/(b – х) = 1/5 * ((b + х)/(х + а)) ↔ 5(х – а)(х + а) = (b + х)(b – х) ↔ 5(х 2 – а 2) = (b 2 – х 2) ↔ 6х 2 = b 2 + 5а 2 ↔ х = √(5а 2 + b 2)/6.

Таким образом, ОЕ = х = √(5а 2 + b 2)/6.

Заключение

Геометрия не самая легкая из наук, но вы наверняка сможете справиться с экзаменационными заданиями. Достаточно проявить немного усидчивости при подготовке. И, конечно, запомнить все нужные формулы.

Мы постарались собрать в одном месте все формулы вычисления площади трапеции, чтобы вы могли воспользоваться ими, когда будете готовиться к экзаменам и повторять материал.

Обязательно расскажите про эту статью одноклассникам и друзьям в социальных сетях. Пускай хороших оценок за ЕГЭ и ГИА будет больше!

сайт, при полном или частичном копировании материала ссылка на первоисточник обязательна.

В математике известно несколько видов четырехугольников: квадрат, прямоугольник, ромб, параллелограмм. Среди них и трапеция — вид выпуклого четырехугольника, у которого две стороны параллельны, а две другие нет. Параллельные противоположные стороны называются основаниями, а две другие – боковыми сторонами трапеции. Отрезок, который соединяет середины боковых сторон, называется средней линией. Существует несколько видов трапеций: равнобедренная, прямоугольная, криволинейная. Для каждого вида трапеции есть формулы для нахождения площади.

Площадь трапеции

Чтобы найти площадь трапеции, нужно знать длину ее оснований и высоту. Высота трапеции — это отрезок, перпендикулярный основаниям. Пусть верхнее основание — a, нижнее основание — b, а высота — h. Тогда вычислить площадь S можно по формуле:

S = ½ * (a+b) * h

т.е. взять полусумму оснований, умноженную на высоту.

Также удастся вычислить площадь трапеции, если известно значение высоты и средней линии. Обозначим среднюю линию — m. Тогда

Решим задачу посложнее: известны длины четырех сторон трапеции — a, b, c, d. Тогда площадь отыщется по формуле:


Если известны длины диагоналей и угол между ними, то площадь ищется так:

S = ½ * d1 * d2 * sin α

где d с индексами 1 и 2 — диагонали. В данной формуле в расчете приводится синус угла.

При известных длинах оснований a и b и двух углах при нижнем основании площадь вычисляется так:

S = ½ * (b2 — a2) * (sin α * sin β / sin(α + β))

Площадь равнобедренной трапеции

Равнобедренная трапеция — это частный случай трапеции. Ее отличие в том, что такая трапеция — это выпуклый четырехугольник с осью симметрии, проходящей через середины двух противоположных сторон. Ее боковые стороны равны.


Найти площадь равнобедренной трапеции можно несколькими способами.

  • Через длины трех сторон. В этом случае длины боковых сторон будут совпадать, поэтому обозначены одной величиной — с, а и b — длины оснований:

  • Если известна длина верхнего основания, боковой стороны и величина угла при нижнем основании, то площадь вычисляется так:

S = c * sin α * (a + c * cos α)

где а — верхнее основание, с — боковая сторона.

  • Если вместо верхнего основания известна длина нижнего – b, площадь рассчитывается по формуле:

S = c * sin α * (b – c * cos α)

  • Если когда известны два основания и угол при нижнем основании, площадь вычисляется через тангенс угла:

S = ½ * (b2 – a2) * tg α

  • Также площадь рассчитывается через диагонали и угол между ними. В этом случае диагонали по длине равны, поэтому каждую обозначаем буквой d без индексов:

S = ½ * d2 * sin α

  • Вычислим площадь трапеции, зная длину боковой стороны, средней линии и величину угла при нижнем основании.

Пусть боковая сторона — с, средняя линия — m, угол — a, тогда:

S = m * c * sin α

Иногда в равностороннюю трапецию можно вписать окружность, радиус которой будет — r.


Известно, что в любую трапецию можно вписать окружность, если сумма длин оснований равна сумме длин ее боковых сторон. Тогда площадь найдется через радиус вписанной окружности и угол при нижнем основании:

S = 4r2 / sin α

Такой же расчет производится и через диаметр D вписанной окружности (кстати, он совпадает с высотой трапеции):

Зная основания и угол, площадь равнобедренной трапеции вычисляется так:

S = a * b / sin α

(эта и последующие формулы верны только для трапеций с вписанной окружностью).


Через основания и радиус окружности площадь ищется так:

Если известны только основания, то площадь считается по формуле:


Через основания и боковую линию площадь трапеции с вписанным кругом и через основания и среднюю линию — m вычисляется так:

Площадь прямоугольной трапеции

Прямоугольной называется трапеция, у которой одна из боковых сторон перпендикулярна основаниям. В этом случае боковая сторона по длине совпадает с высотой трапеции.

Прямоугольная трапеция представляет из себя квадрат и треугольник. Найдя площадь каждой из фигур, сложите полученные результаты и получите общую площадь фигуры.


Также для вычисления площади прямоугольной трапеции подходят общие формулы для расчета площади трапеции.

  • Если известны длины оснований и высота (или перпендикулярная боковая сторона), то площадь рассчитывается по формуле:

S = (a + b) * h / 2

В качестве h (высоты) может выступать боковая сторона с. Тогда формула выглядит так:

S = (a + b) * c / 2

  • Другой способ рассчитать площадь — перемножить длину средней линии на высоту:

или на длину боковой перпендикулярной стороны:

  • Следующий способ вычисления — через половину произведения диагоналей и синус угла между ними:

S = ½ * d1 * d2 * sin α


Если диагонали перпендикулярны, то формула упрощается до:

S = ½ * d1 * d2

  • Еще один способ вычисления — через полупериметр (сумма длин двух противоположных сторон) и радиус вписанной окружности.

Эта формула действительна для оснований. Если брать длины боковых сторон, то одна из них будет равна удвоенному радиусу. Формула будет выглядеть так:

S = (2r + c) * r

  • Если в трапецию вписана окружность, то площадь вычисляется так же:

где m — длина средней линии.

Площадь криволинейной трапеции

Криволинейная трапеция представляет из себя плоскую фигуру, ограниченную графиком неотрицательной непрерывной функции y = f(x), определенной на отрезке , осью абсцисс и прямыми x = a, x = b. По сути, две ее стороны параллельны друг другу (основания), третья сторона перпендикулярна основаниям, а четвертая представляет из себя кривую, соответствующую графику функции.


Площадь криволинейной трапеции ищут через интеграл по формуле Ньютона-Лейбница:


Так вычисляются площади различных видов трапеций. Но, помимо свойств сторон, трапеции обладают одинаковыми свойствами углов. Как у всех существующих четырехугольников, сумма внутренних углов трапеции равна 360 градусов. А сумма углов, прилежащих к боковой стороне, — 180 градусам.

Рекомендуем также

Как найти высоту трапеции через диагонали. Как найти площадь трапеции: формулы и примеры

Практика прошлогодних ЕГЭ и ГИА показывает, что задачи по геометрии вызывают сложности у многих школьников. Вы легко справитесь с ними, если заучите все нужные формулы и попрактикуетесь в решении задач.

В этой статье вы увидите формулы нахождения площади трапеции, а также примеры задач с решениями. Такие же могут попасться вам в КИМах на аттестационных экзаменах или на олимпиадах. Поэтому отнеситесь к ним внимательно.

Что нужно знать про трапецию?

Для начала вспомним, что трапецией называется четырехугольник, у которого две противоположные стороны, их еще называют основаниями, параллельны, а две другие – нет.

В трапеции также может быть опущена высота (перпендикуляр к основанию). Проведена средняя линия – это прямая, которая параллельна основаниям и равна половине их суммы. А также диагонали, которые могут пересекаться, образуя острые и тупые углы. Или, в отдельных случаях, под прямым углом. Кроме того, если трапеция равнобедренная, в нее можно вписать окружность. И описать окружность около нее.

Формулы площади трапеции

Для начала рассмотрим стандартные формулы нахождения площади трапеции. Способы вычислить площадь равнобедренной и криволинейной трапеций рассмотрим ниже.

Итак, представьте, что у вас есть трапеция с основаниями a и b, в которой к большему основанию опущена высота h. Вычислить площадь фигуры в таком случае проще простого. Надо всего лишь разделить на два сумму длин оснований и умножить то, что получится, на высоту: S = 1/2(a + b)*h .

Возьмем другой случай: предположим, в трапеции, кроме высоты, проведена средняя линия m. Нам известна формула нахождения длины средней линии: m = 1/2(a + b). Поэтому с полным правом можем упростить формулу площади трапеции до следующего вида: S = m* h . Другими словами, чтобы найти площадь трапеции, надо умножить среднюю линию на высоту.

Рассмотрим еще один вариант: в трапеции проведены диагонали d 1 и d 2 , которые пересекаются не под прямым углом α. Чтобы вычислить площадь такой трапеции, вам нужно разделить на два произведение диагоналей и умножить то, что получится, на sin угла между ними: S= 1/2d 1 d 2 *sinα .

Теперь рассмотрим формулу для нахождения площади трапеции, если о ней неизвестно ничего, кроме длин всех ее сторон: a, b, c и d. Это громоздкая и сложная формула, но вам будет полезно запомнить на всякий случай и ее: S = 1/2(a + b) * √c 2 – ((1/2(b – a)) * ((b – a) 2 + c 2 – d 2)) 2 .

Кстати, приведенные выше примеры верны и для того случая, когда вам потребуется формула площади прямоугольной трапеции. Эта трапеция, боковая сторона которой примыкает к основаниям под прямым углом.

Равнобедренная трапеция

Трапеция, боковые стороны которой равны, называется равнобедренной. Мы рассмотрим несколько вариантов формулы площади равнобедренной трапеции.

Первый вариант: для случая, когда внутрь равнобедренной трапеции вписана окружность с радиусом r, а боковая сторона и большее основание образуют острый угол α. Окружность может быть вписана в трапецию при условии, что сумма длин ее оснований равна сумме длин боковых сторон.

Площадь равнобедренной трапеции вычисляется так: умножьте квадрат радиуса вписанной окружности на четыре и разделите все это на sinα: S = 4r 2 /sinα . Еще одна формула площади является частным случаем для того варианта, когда угол между большим основанием и боковой стороной равен 30 0: S = 8r 2 .

Второй вариант: на этот раз возьмем равнобедренную трапецию, в которой вдобавок проведены диагонали d 1 и d 2 , а также высота h. Если диагонали трапеции взаимно перпендикулярны, высота составляет половину суммы оснований: h = 1/2(a + b). Зная это, легко преобразовать уже знакомую вам формулу площади трапеции в такой вид: S = h 2 .

Формула площади криволинейной трапеции

Начнем с того, что разберемся: что такое криволинейная трапеция. Представьте себе ось координат и график непрерывной и неотрицательной функции f, которая не меняет знака в пределах заданного отрезка на оси x. Криволинейную трапецию образуют график функции у = f(x) – вверху, ось х – внизу (отрезок ), а по бокам – прямые, проведенные между точками a и b и графиком функции.

Вычислить площадь такой нестандартной фигуры нельзя приведенными выше способами. Тут нужно применить математический анализ и использовать интеграл. А именно: формулу Ньютона-Лейбница – S = ∫ b a f(x)dx = F(x)│ b a = F(b) – F(a) . В этой формуле F – первообразная нашей функции на выбранном отрезке . И площадь криволинейной трапеции соответствует приращению первообразной на заданном отрезке.

Примеры задач

Чтобы все эти формулы лучше улеглись в голове, вот вам несколько примеров задач на нахождение площади трапеции. Лучше всего будет, если вы сперва попробуете решить задачи сами, и только потом сверите полученный ответ с готовым решением.

Задача №1: Дана трапеция. Ее большее основание – 11 см, меньшее – 4см. В трапеции проведены диагонали, одна длиной 12 см, вторая – 9 см.

Решение: Постройте трапецию АМРС. Проведите прямую РХ через вершину Р так, чтобы она оказалась параллельной диагонали МС и пересекла прямую АС в точке Х. Получится треугольник АРХ.

Мы рассмотрим две полученных в результате этих манипуляций фигуры: треугольник АРХ и параллелограмм СМРХ.

Благодаря параллелограмму мы узнаем, что РХ = МС = 12 см и СХ = МР = 4см. Откуда можем вычислить сторону АХ треугольника АРХ: АХ = АС + СХ = 11 + 4 = 15 см.

Мы также можем доказать, что треугольник АРХ – прямоугольный (для этого примените теорему Пифагора – АХ 2 = АР 2 + РХ 2). И высчитать его площадь: S APX = 1/2(AP * PX) = 1/2(9 * 12) = 54 см 2 .

Дальше вам потребуется доказать, что треугольники АМР и РСХ являются равновеликими. Основанием послужит равенство сторон МР и СХ (уже доказанное выше). А также высоты, которые вы опустите на эти стороны – они равны высоте трапеции АМРС.

Все это позволит вам утверждать, что S AMPC = S APX = 54 см 2 .

Задача №2: Дана трапеция КРМС. На ее боковых сторонах расположены точки О и Е, при этом ОЕ и КС параллельны. Также известно, что площади трапеций ОРМЕ и ОКСЕ находятся в соотношении 1:5. РМ = а и КС = b. Требуется найти ОЕ.

Решение: Проведите через точку М прямую, параллельную РК, и точку ее пересечения с ОЕ обозначьте Т. А – точка пересечения прямой, проведенной через точку Е параллельно РК, с основанием КС.

Введем еще одно обозначение – ОЕ = х. А также высоту h 1 для треугольника ТМЕ и высоту h 2 для треугольника АЕС (вы можете самостоятельно доказать подобие этих треугольников).

Будем считать, что b > а. Площади трапеций ОРМЕ и ОКСЕ относятся как 1:5, что дает нам право составить такое уравнение: (х + а) * h 1 = 1/5(b + х) * h 2 . Преобразуем и получим: h 1 / h 2 = 1/5 * ((b + х)/(х + а)).

Раз треугольники ТМЕ и АЕС подобные, имеем h 1 / h 2 = (х – а)/(b – х). Объединим обе записи и получим: (х – а)/(b – х) = 1/5 * ((b + х)/(х + а)) ↔ 5(х – а)(х + а) = (b + х)(b – х) ↔ 5(х 2 – а 2) = (b 2 – х 2) ↔ 6х 2 = b 2 + 5а 2 ↔ х = √(5а 2 + b 2)/6.

Таким образом, ОЕ = х = √(5а 2 + b 2)/6.

Заключение

Геометрия не самая легкая из наук, но вы наверняка сможете справиться с экзаменационными заданиями. Достаточно проявить немного усидчивости при подготовке. И, конечно, запомнить все нужные формулы.

Мы постарались собрать в одном месте все формулы вычисления площади трапеции, чтобы вы могли воспользоваться ими, когда будете готовиться к экзаменам и повторять материал.

Обязательно расскажите про эту статью одноклассникам и друзьям в социальных сетях. Пускай хороших оценок за ЕГЭ и ГИА будет больше!

сайт, при полном или частичном копировании материала ссылка на первоисточник обязательна.

На простой вопрос «Как найти высоту трапеции?» существует несколько ответов, и все потому, что могут быть даны разные исходные величины. Поэтому и формулы будут различаться.

Эти формулы можно запомнить, но они несложно выводятся. Нужно только применять ранее изученные теоремы.

Принятые в формулах обозначения

Во всех приведенных ниже математических записях верны такие прочтения букв.

В исходных данных: все стороны

Для того чтобы найти высоту трапеции в общем случае потребуется воспользоваться такой формулой:

н = √(с 2 — (((а — в) 2 + с 2 — d 2)/(2(а — в))) 2). Номер 1.

Не самая короткая, но и встречается в задачах достаточно редко. Обычно можно воспользоваться другими данными.

Формула, которая подскажет, как найти высоту равнобедренной трапеции в той же ситуации, гораздо короче:

н = √(с 2 — (а — в) 2 /4). Номер 2.

В задаче даны: боковые стороны и углы при нижнем основании

Принимают, что угол α прилежит к боковой стороне с обозначением «с», соответственно угол β к стороне d. Тогда формула для того, как найти высоту трапеции, в общем виде будет такой:

н = с * sin α= d * sin β. Номер 3.

Если фигура равнобедренная, то можно воспользоваться таким вариантом:

н = с * sin α= ((а — в) / 2) * tg α. Номер 4.

Известны: диагонали и углы между ними

Обычно к этим данным присоединяются еще известные величины. Например, основания или средняя линия. Если даны основания, то для ответа на вопрос, как найти высоту трапеции, пригодится такая формула:

н = (d 1 * d 2 * sin γ) / (а + в) или н = (d 1 * d 2 * sin δ) / (а + в). Номер 5.

Это для общего вида фигуры. Если дана равнобедренная, то запись преобразится так:

н = (d 1 2 * sin γ) / (а + в) или н = (d 1 2 * sin δ) / (а + в). Номер 6.

Когда в задаче идет речь о средней линии трапеции, то формулы для поиска ее высоты становятся такими:

н = (d 1 * d 2 * sin γ) / 2m или н = (d 1 * d 2 * sin δ) / 2m. Номер 5а.

н = (d 1 2 * sin γ) / 2m или н = (d 1 2 * sin δ) / 2m. Номер 6а.

Среди известных величин: площадь с основаниями или средней линией

Это, пожалуй, самые короткие и простые формулы того, как найти высоту трапеции. Для произвольной фигуры она будет такой:

н = 2S / (а + в). Номер 7.

Она же, но с известной средней линией:

н = S / m. Номер 7а.

Как ни странно, но для равнобедренной трапеции формулы будут выглядеть так же.

Задачи

№1. На определение углов при нижнем основании трапеции.

Условие. Дана равнобедренная трапеция, боковая сторона которой 5 см. Ее основания равны 6 и 12 см. Требуется найти синус острого угла.

Решение. Для удобства следует ввести обозначение. Пусть левая нижняя вершина будет А, все остальные по часовой стрелке: В, С, Д. Таким образом, нижнее основание будет обозначено АД, верхнее — ВС.

Нужно провести высоты из вершин В и С. Точки, которые укажут концы высот будут обозначены Н 1 и Н 2 , соответственно. Поскольку в фигуре ВСН 1 Н 2 все углы прямые, то она является прямоугольником. Это означает, что отрезок Н 1 Н 2 равен 6 см.

Теперь нужно рассмотреть два треугольника. Они равны, так как являются прямоугольными с одинаковыми гипотенузами и вертикальными катетами. Отсюда следует, что и меньшие катеты у них равны. Поэтому их можно определить как частное от разности. Последняя получится от вычитания из нижнего основания верхнего. Делиться оно будет на 2. То есть 12 — 6 нужно поделить на 2. АН 1 = Н 2 Д = 3 (см).

Теперь из теоремы Пифагора нужно найти высоту трапеции. Она необходима для нахождения синуса угла. ВН 1 = √(5 2 — 3 2) = 4 (см).

Воспользовавшись знанием о том, как находится синус острого угла в треугольнике с прямым углом, можно записать такое выражение: sin α= ВН 1 / АВ = 0,8.

Ответ. Искомый синус равен 0,8.

№2. На нахождение высоты трапеции по известному тангенсу.

Условие. У равнобедренной трапеции нужно вычислить высоту. Известно, что ее основания равны 15 и 28 см. Дан тангенс острого угла: 11/13.

Решение. Обозначение вершин такое же, как в предыдущей задаче. Снова нужно провести две высоты из верхних углов. По аналогии с решением первой задачи нужно найти АН 1 = Н 2 Д, которые определятся как разность 28 и 15, деленная на два. После подсчетов получается: 6,5 см.

Поскольку тангенс — это отношение двух катетов, то можно записать такое равенство: tg α= АН 1 / ВН 1 . Причем это отношение равно 11/13 (по условию). Так как АН 1 известен, то можно вычислить высоту: ВН 1 = (11 * 6,5) / 13. Простые расчеты дают результат в 5,5 см.

Ответ. Искомая высота равна 5,5 см.

№3. На вычисление высоты по известным диагоналям.

Условие. О трапеции известно, что ее диагонали равны 13 и 3 см. Нужно узнать ее высоту, если сумма оснований составляет 14 см.

Решение. Пусть обозначение фигуры будет таким же, как раньше. Предположим, что АС — меньшая диагональ. Из вершины С нужно провести искомую высоту и обозначить ее СН.

Теперь потребуется выполнить дополнительное построение. Из угла С нужно провести прямую, параллельную большей диагонали и найти точку ее пересечения с продолжением стороны АД. Это будет Д 1 . Получилась новая трапеция, внутри которой начерчен треугольник АСД 1 . Он-то и нужен для дальнейшего решения задачи.

Искомая высота окажется еще и ей же в треугольнике. Поэтому можно воспользоваться формулами, изученными в другой теме. Высота треугольника определяется как произведение числа 2 и площади, деленное на сторону, к которой она проведена. А сторона оказывается равна сумме оснований исходной трапеции. Это исходит из правила, по которому выполнено дополнительное построение.

В рассматриваемом треугольнике все стороны известны. Для удобства введем обозначения х = 3 см, у = 13 см, z = 14 см.

Теперь можно сосчитать площадь, воспользовавшись теоремой Герона. Полупериметр будет равен р = (х + у + z)/ 2 = (3 + 13 + 14) / 2 = 15 (см). Тогда формула для площади после подстановки значений будет выглядеть так: S = √(15 * (15 — 3) * (15 — 13) * (15 — 14)) = 6 √10 (см 2).

Ответ. Высота равна 6√10 / 7 см.

№4. Для поиска высоты по сторонам.

Условие. Дана трапеция, три стороны которой равны 10 см, а четвертая 24 см. Нужно узнать ее высоту.

Решение. Поскольку фигура равнобедренная, то потребуется формула под номером 2. В нее нужно просто подставить все значения и сосчитать. Это будет выглядеть так:

н = √(10 2 — (10 — 24) 2 /4) = √51 (см).

Ответ. н = √51 см.

Многоликая трапеция… Она может быть произвольной, равнобедренной или прямоугольной. И в каждом случае нужно знать, как найти площадь трапеции. Конечно, проще всего запомнить основные формулы. Но иногда проще воспользоваться той, которая выведена с учетом всех особенностей конкретной геометрической фигуры.

Несколько слов о трапеции и ее элементах

Любой четырехугольник, у которого две стороны параллельны, можно назвать трапецией. В общем случае они не равны и называются основаниями. Большее из них — нижнее, а другое — верхнее.

Две другие стороны оказываются боковыми. У произвольной трапеции они имеют различную длину. Если же они равны, то фигура становится равнобедренной.

Если вдруг угол между любой боковой стороной и основанием окажется равным 90 градусам, то трапеция является прямоугольной.

Все эти особенности могут помочь в решении задачи о том, как найти площадь трапеции.

Среди элементов фигуры, которые могут оказаться незаменимыми в решении задач, можно выделить такие:

  • высота, то есть отрезок, перпендикулярный обоим основаниям;
  • средняя линия, которая имеет своими концами середины боковых сторон.

По какой формуле вычислить площадь, если известны основания и высота?

Это выражение дается основным, потому что чаще всего можно узнать эти величины, даже когда они не даны явно. Итак, чтобы понять, как найти площадь трапеции, потребуется сложить оба основания и разделить их на два. Получившееся значение потом еще умножить на значение высоты.

Если обозначить основания буквами а 1 и а 2 , высоту — н, то формула для площади будет выглядеть так:

S = ((а 1 + а 2)/2)*н.

Формула, по которой вычисляется площадь, если даны ее высота и средняя линия

Если посмотреть внимательно на предыдущую формулу, то легко заметить, что в ней явно присутствует значение средней линии. А именно, сумма оснований, деленная на два. Пусть средняя линия будет обозначена буквой l, тогда формула для площади станет такой:

S = l * н.

Возможность найти площадь по диагоналям

Этот способ поможет, если известен угол, образованный ими. Предположим, что диагонали обозначены буквами д 1 и д 2 , а углы между ними — α и β. Тогда формула того, как найти площадь трапеции, будет записана следующим образом:

S = ((д 1 * д 2)/2) * sin α.

В этом выражении можно легко заменить α на β. Результат не изменится.

Как узнать площадь, если известны все стороны фигуры?

Бывают и такие ситуации, когда в этой фигуре известны именно стороны. Эта формула получается громоздкой и ее сложно запомнить. Но возможно. Пусть боковые стороны имеют обозначение: в 1 и в 2 , основание а 1 больше, чем а 2 . Тогда формула площади примет такой вид:

S = ((а 1 + а 2) / 2) * √ {в 1 2 — [(а 1 — а 2) 2 + в 1 2 — в 2 2) / (2 * (а 1 — а 2))] 2 }.

Способы вычисления площади равнобедренной трапеции

Первый связан с тем, что в нее можно вписать окружность. И, зная ее радиус (он обозначается буквой r), а также угол при основании — γ, можно воспользоваться такой формулой:

S = (4 * r 2) / sin γ.

Последняя общая формула, которая основана на знании всех сторон фигуры, существенно упростится за счет того, что боковые стороны имеют одинаковое значение:

S = ((а 1 + а 2) / 2) * √ {в 2 — [(а 1 — а 2) 2 / (2 * (а 1 — а 2))] 2 }.

Методы вычисления площади прямоугольной трапеции

Понятно, что подойдет любой из перечисленных для произвольной фигуры. Но иногда полезно знать об одной особенности такой трапеции. Она заключается в том, что разность квадратов длин диагоналей равна разности, составленной из квадратов оснований.

Часто формулы для трапеции забываются, в то время как выражения для площадей прямоугольника и треугольника помнятся. Тогда можно применить простой способ. Разделить трапецию на две фигуры, если она прямоугольная, или три. Одна точно будет прямоугольником, а вторая, или две оставшиеся, треугольниками. После вычисления площадей этих фигур останется их только сложить.

Это достаточно простой способ того, как найти площадь прямоугольной трапеции.

Как быть, если известны координаты вершин трапеции?

В этом случае потребуется воспользоваться выражением, которое позволяет определить расстояние между точками. Его можно применить три раза: для того, чтобы узнать оба основания и одну высоту. А потом просто применить первую формулу, которая описана немного выше.

Для иллюстрации такого метода можно привести такой пример. Даны вершины с координатами А(5; 7), В(8; 7), С(10; 1), Д(1; 1). Нужно узнать площадь фигуры.

До того как найти площадь трапеции, по координатам нужно вычислить длины оснований. Потребуется такая формула:

длина отрезка = √{(разность первых координат точек) 2 + (разность вторых координат точек) 2 }.

Верхнее основание обозначено АВ, значит, его длина будет равна √{(8-5) 2 + (7-7) 2 } = √9 = 3. Нижнее — СД = √ {(10-1) 2 + (1-1) 2 } = √81 = 9.

Теперь нужно провести высоту из вершины на основание. Пусть ее начало будет в точке А. Конец отрезка окажется на нижнем основании в точке с координатами (5; 1), пусть это будет точка Н. Длина отрезка АН получится равной √{(5-5) 2 + (7-1) 2 } = √36 = 6.

Осталось только подставить получавшиеся значения в формулу площади трапеции:

S = ((3 + 9) / 2) * 6 = 36.

Задача решена без единиц измерения, потому что не указан масштаб координатной сетки. Он может быть как миллиметр, так и метр.

Примеры задач

№ 1. Условие. Известен угол между диагоналями произвольной трапеции, он равен 30 градусам. Меньшая диагональ имеет значение 3 дм, а вторая больше ее в 2 раза. Необходимо посчитать площадь трапеции.

Решение. Для начала нужно узнать длину второй диагонали, потому что без этого не удастся сосчитать ответ. Вычислить ее несложно, 3 * 2 = 6 (дм).

Теперь нужно воспользоваться подходящей формулой для площади:

S = ((3 * 6) / 2) * sin 30º = 18/2 * ½ = 4,5 (дм 2). Задача решена.

Ответ: площадь трапеции равна 4,5 дм 2 .

№ 2. Условие. В трапеции АВСД основаниями являются отрезки АД и ВС. Точка Е — середина стороны СД. Из нее проведен перпендикуляр к прямой АВ, конец этого отрезка обозначен буквой Н. Известно, что длины АВ и ЕН равны соответственно 5 и 4 см. Нужно вычислить площадь трапеции.

Решение. Для начала нужно сделать чертеж. Поскольку значение перпендикуляра меньше стороны, к которой он проведен, то трапеция будет немного вытянутой вверх. Так ЕН окажется внутри фигуры.

Чтобы отчетливо увидеть ход решения задачи, потребуется выполнить дополнительное построение. А именно, провести прямую, которая будет параллельна стороне АВ. Точки пересечения этой прямой с АД — Р, а с продолжением ВС — Х. Получившаяся фигура ВХРА — параллелограмм. Причем его площадь равна искомой. Это связано с тем, что треугольники, которые получились при дополнительном построении, равны. Это следует из равенства стороны и двух прилежащих к ней углов, один — вертикальный, другой — накрест лежащий.

Найти площадь параллелограмма можно по формуле, которая содержит произведение стороны и высоты, опущенной на нее.

Таким образом, площадь трапеции равна 5 * 4 = 20 см 2 .

Ответ: S = 20 см 2 .

№ 3. Условие. Элементы равнобедренной трапеции имеют такие значения: нижнее основание — 14 см, верхнее — 4 см, острый угол — 45º. Нужно вычислить ее площадь.

Решение. Пусть меньшее основание имеет обозначение ВС. Высота, проведенная из точки В, будет называться ВН. Поскольку угол 45º, то треугольник АВН получится прямоугольный и равнобедренный. Значит, АН=ВН. Причем АН очень легко найти. Она равна половине разности оснований. То есть (14 — 4) / 2 = 10 / 2 = 5 (см).

Основания известны, высота сосчитана. Можно пользоваться первой формулой, которая здесь была рассмотрена для произвольной трапеции.

S = ((14 + 4) / 2) * 5 = 18/2 * 5 = 9 * 5 = 45 (см 2).

Ответ: Искомая площадь равна 45 см 2 .

№ 4. Условие. Имеется произвольная трапеция АВСД. На ее боковых сторонах взяты точки О и Е, так что ОЕ параллельна основанию АД. Площадь трапеции АОЕД в пять раз больше, чем у ОВСЕ. Вычислить значение ОЕ, если известны длины оснований.

Решение. Потребуется провести две параллельные АВ прямые: первую через точку С, ее пересечение с ОЕ — точка Т; вторую через Е и точкой пересечения с АД будет М.

Пусть неизвестная ОЕ=х. Высота меньшей трапеции ОВСЕ — н 1 , большей АОЕД — н 2 .

Поскольку площади этих двух трапеций соотносятся как 1 к 5, то можно записать такое равенство:

(х + а 2) * н 1 = 1/5 (х + а 1) * н 2

н 1 /н 2 = (х + а 1) / (5(х + а 2)).

Высоты и стороны треугольников пропорциональны по построению. Поэтому можно записать еще одно равенство:

н 1 /н 2 = (х — а 2) / (а 1 — х).

В двух последних записях в левой части стоят равные величины, значит, можно написать, что (х + а 1) / (5(х + а 2)) равно (х — а 2) / (а 1 — х).

Здесь требуется провести ряд преобразований. Сначала перемножить крест накрест. Появятся скобки, которые укажут на разность квадратов, после применения этой формулы получится короткое уравнение.

В нем нужно раскрыть скобки и перенести все слагаемые с неизвестной «х» в левую сторону, а потом извлечь квадратный корень.

Ответ : х = √ {(а 1 2 + 5 а 2 2) / 6}.

(S) трапеции, начните вычисление высоты (h) с нахождения полусуммы длин параллельных сторон: (a+b)/2. Затем на полученное значение разделите площадь — результат и будет искомой величиной: h = S/((a+b)/2) = 2*S/(a+b).

Зная длину средней линии (m) и площадь (S) можно упростить формулу из предыдущего шага. По определению средняя линия трапеции равна полусумме ее оснований, поэтому для вычисления высоты (h) фигуры просто разделите площадь на длину средней линии: h = S/m.

Можно определить высоту (h) такого и в том случае, если даны только длина одной из боковых сторон (с) и угол (α), образуемый ей и длинным основанием. В этом случае следует рассмотреть , образуемый этой стороной, высотой и коротким отрезком основания, который отсекает опущенная на него высота. Этот треугольник будет прямоугольным, известная сторона будет в нем гипотенузой, а высота — катетом. Отношение длин и гипотенузы равно противолежащего катету угла, поэтому для вычисления высоты трапеции умножьте известную длину стороны на синус известного угла: h = с*sin(α).

Такой же треугольник стоит рассмотреть и если даны длина боковой стороны (с) и величина угла (β) между ней и другим (коротким) основанием. В этом случае величина угла между боковой стороной (гипотенузой) и высотой (катетом) будет на 90° меньше известного из условий угла: β-90°. Так как отношение длин катета и гипотенузы равно косинусу угла между ними, то высоту трапеции вычислите умножением косинуса уменьшенного на 90° угла на длину боковой стороны: h = с*cos(β-90°).

Если вписана окружность известного радиуса (r), вычисления высоты (h) будет очень проста и не потребует никаких других параметров. Такая окружность по определению должна каждого из оснований только одной точкой и эти точки будут лежать на одной линии с центром . Это значит, что расстояние между ними будет равно диаметру (удвоенному радиусу), проведенному перпендикулярно основаниям, то есть совпадающим с высотой трапеции: h=2*r.

Трапецией считается такой четырехугольник, у которого две стороны параллельны, а две другие нет. Высотой трапеции называется отрезок, проведенный перпендикулярно между двумя параллельными прямыми. В зависимости от исходных данных ее можно вычислить по-разному.

Вам понадобится

  • Знание сторон, оснований, средней линии трапеции, а так же, опционально, ее площадь и/или периметр.

Инструкция

Допустим, имеется трапеция с теми же данными, что и на рисунке 1. Проведем 2 высоты, получим , у которого 2 меньшие стороны катетами прямоугольных треугольников. Обозначим меньший катит за x. Он находится

Трапецией называется четырехугольник, у которого только две стороны параллельны между собой.

Они называются основаниями фигуры, оставшиеся – боковыми сторонами. Частными случаями фигуры считается параллелограмм. Также существует криволинейная трапеция, которая включает в себя график функции. Формулы площади трапеции включают в себя практически все ее элементы, и лучшее решение подбирается в зависимости от заданных величин.
Основные роли в трапеции отводятся высоте и средней линии. Средняя линия – это линия, соединяющая середины боковых сторон. Высота трапеции проводится под прямым углом от верхнего угла к основанию.
Площадь трапеции через высоту равняется произведению полусуммы длин оснований, умноженному на высоту:

Если по условиям известна средняя линия, то эта формула значительно упрощается, так как она равна полусумме длин оснований :

Если по условиям даны длины всех сторон, то можно рассмотреть пример расчета площади трапеции через эти данные:

Допустим, дана трапеция с основаниями a = 3 см, b = 7 см и боковыми сторонами c = 5 см, d = 4 см. найдем площадь фигуры:

Площадь равнобокой трапеции


Отдельным случаем считается равнобокая или, как ее еще называют, равнобедренная трапеция.
Особым случаем является и нахождение площади равнобедренной (равнобокой) трапеции. Формула выводится различными способами – через диагонали, через углы, прилегающие к основанию и радиус вписанной окружности.
Если по условиям задана длина диагоналей и известен угол между ними можно использовать такую формулу:

Помните, что диагонали равнобокой трапеции равны между собой!


То есть, зная одно их оснований, сторону и угол, можно легко рассчитать площадь.

Площадь криволинейной трапеции


Отдельный случай – это криволинейная трапеция . Она располагается на оси координат и ограничивается графиком непрерывной положительной функции.

Ее основание располагает на оси X и ограничивается двумя точками:
Интегралы помогают вычислить площадь криволинейной трапеции.
Формула прописывается так:

Рассмотрим пример расчета площади криволинейной трапеции. Формула требует определенных знаний для работы с определенными интегралами. Для начала разберем значение определенного интеграла:

Здесь F(a) – это значение первообразной функции f(x) в точке a , F(b) – значение этой же функции f(x) в точке b .

Теперь решим задачу. На рисунке изображена криволинейная трапеция, ограниченная функцией . Функция
Нам необходимо найти площадь выделенной фигуры, которая является криволинейной трапецией, ограниченной сверху графиком , справа прямой x ={-8}, слева прямой x ={-10} и осью OX снизу.
Площадь этой фигуры мы будем рассчитывать по формуле:

Условиями задачи нам задана функция. По ней мы найдем значения первообразной в каждой из наших точек:

Теперь
Ответ: площадь заданной криволинейной трапеции равняется 4.

Ничего сложного в расчетах этого значения нет. Важна только предельная внимательность в вычислениях.

Формула площади трапеции

В статье понятно и доступно разберем формулу площади трапеции, но для начала отработаем основные понятия!
Трапеция это геометрическая четырехугольная фигура, состоящая из двух параллельных линий называющихся основанием и двух боковых линии не являющиеся параллельными, называющиеся боковые стороны. Линия которая соединяет стороны как основные так и боковые посередине, называется — средней линией, высота выводится под углом 900.
Площадью трапеции называется участок на плоскости, который ограничен данной фигурой, обозначается в единицах квадратных.


В случае если мы знаем величину средней линии k, формула меняется на более легкую, она приравнивается к половине суммы длины основных линий

В случае когда мы знаем длину всех сторон, можно рассчитать площадь используя данную формулу

Если разобрать данную формулу на примере, то мы получим следующее:
Рассмотрим для ясности: трапеция с длиной боковых линий х = 5 см, g = 4 см, основные линии y = 3 см, z = 7 см. Требуется найти S = ?.
Трапеция бывает однобокой, ещё ее называют равнобедренной — так как диагонали равны между собой. Для нее формула может складываться через радиус вписанной в нее окружности, диагонали и углы прилегающие к основанию.
В случае когда мы знаем длину диагоналей и угол находящийся между ними:

В том случае когда выводим формулу с помощью боковых сторон и углов прилегающих к основанию. Формула будет выглядеть так:

S = x * sin ?(y — x * cos ?)
S = x * sin ?(z + x * cos ?)

Вывод: Если нам известно одно основание из двух и величины углов принадлежащие этому основанию, мы без труда сможем узнать площадь трапеции.

Трапеция бывает криволинейной — это тогда, когда трапеция находится на оси координат, ограничена графиком продолжительной функции.
В случае когда основание трапеции находится на оси х и ограничено точками x1 = z, x2 = y. Вычислить площадь трапеции помогут интегралы



где F (z) — значение в точке z
F (y) — значение в точке y

Разберем для наглядности: Криволинейная трапеция, ограниченная функцией y = f(x). Функция F(x) = — x3 — 27×2 — 240x — 8. Нужно найти S = ?. Фигура ограничивается: графиком сверху y = f(x)., снизу ОХ осью, слева х = (-10), справа х = (-8).
Пользуемся данной формулой, получаем:


В условиях задачи дана функция. С помощью нее найдем значения точек.
1) F(-8) = -(-8)3 — 27 х (-8)2 — 240 х (-8) — 8 = 24-1728+1920 — 8 = 696
2) F(-10) = -(-10)3 — 27 х (-10)2 — 240 х (-10) — 8 = 1000-2700+240 — 8 = 692
3) F(-8) — F (-10) = 696 — 692 = 4
Ответ: S = 4

Вот собственно и всё по формулам площади для разных видов трапеций. Если у вас появились какие то вопросы, обязательно пишите их в комментариях. Успехов в учебе.
vamsochinenie.ru — база сочинений на самые разные темы.


Если материал был полезен, вы можете отправить донат или поделиться данным материалом в социальных сетях:

формулы на все случаи жизни. Площадь трапеции

В математике известно несколько видов четырехугольников: квадрат, прямоугольник, ромб, параллелограмм. Среди них и трапеция — вид выпуклого четырехугольника, у которого две стороны параллельны, а две другие нет. Параллельные противоположные стороны называются основаниями, а две другие – боковыми сторонами трапеции. Отрезок, который соединяет середины боковых сторон, называется средней линией. Существует несколько видов трапеций: равнобедренная, прямоугольная, криволинейная. Для каждого вида трапеции есть формулы для нахождения площади.

Площадь трапеции

Чтобы найти площадь трапеции, нужно знать длину ее оснований и высоту. Высота трапеции — это отрезок, перпендикулярный основаниям. Пусть верхнее основание — a, нижнее основание — b, а высота — h. Тогда вычислить площадь S можно по формуле:

S = ½ * (a+b) * h

т.е. взять полусумму оснований, умноженную на высоту.

Также удастся вычислить площадь трапеции, если известно значение высоты и средней линии. Обозначим среднюю линию — m. Тогда

Решим задачу посложнее: известны длины четырех сторон трапеции — a, b, c, d. Тогда площадь отыщется по формуле:


Если известны длины диагоналей и угол между ними, то площадь ищется так:

S = ½ * d1 * d2 * sin α

где d с индексами 1 и 2 — диагонали. В данной формуле в расчете приводится синус угла.

При известных длинах оснований a и b и двух углах при нижнем основании площадь вычисляется так:

S = ½ * (b2 — a2) * (sin α * sin β / sin(α + β))

Площадь равнобедренной трапеции

Равнобедренная трапеция — это частный случай трапеции. Ее отличие в том, что такая трапеция — это выпуклый четырехугольник с осью симметрии, проходящей через середины двух противоположных сторон. Ее боковые стороны равны.


Найти площадь равнобедренной трапеции можно несколькими способами.

  • Через длины трех сторон. В этом случае длины боковых сторон будут совпадать, поэтому обозначены одной величиной — с, а и b — длины оснований:

  • Если известна длина верхнего основания, боковой стороны и величина угла при нижнем основании, то площадь вычисляется так:

S = c * sin α * (a + c * cos α)

где а — верхнее основание, с — боковая сторона.

  • Если вместо верхнего основания известна длина нижнего – b, площадь рассчитывается по формуле:

S = c * sin α * (b – c * cos α)

  • Если когда известны два основания и угол при нижнем основании, площадь вычисляется через тангенс угла:

S = ½ * (b2 – a2) * tg α

  • Также площадь рассчитывается через диагонали и угол между ними. В этом случае диагонали по длине равны, поэтому каждую обозначаем буквой d без индексов:

S = ½ * d2 * sin α

  • Вычислим площадь трапеции, зная длину боковой стороны, средней линии и величину угла при нижнем основании.

Пусть боковая сторона — с, средняя линия — m, угол — a, тогда:

S = m * c * sin α

Иногда в равностороннюю трапецию можно вписать окружность, радиус которой будет — r.


Известно, что в любую трапецию можно вписать окружность, если сумма длин оснований равна сумме длин ее боковых сторон. Тогда площадь найдется через радиус вписанной окружности и угол при нижнем основании:

S = 4r2 / sin α

Такой же расчет производится и через диаметр D вписанной окружности (кстати, он совпадает с высотой трапеции):

Зная основания и угол, площадь равнобедренной трапеции вычисляется так:

S = a * b / sin α

(эта и последующие формулы верны только для трапеций с вписанной окружностью).


Через основания и радиус окружности площадь ищется так:

Если известны только основания, то площадь считается по формуле:


Через основания и боковую линию площадь трапеции с вписанным кругом и через основания и среднюю линию — m вычисляется так:

Площадь прямоугольной трапеции

Прямоугольной называется трапеция, у которой одна из боковых сторон перпендикулярна основаниям. В этом случае боковая сторона по длине совпадает с высотой трапеции.

Прямоугольная трапеция представляет из себя квадрат и треугольник. Найдя площадь каждой из фигур, сложите полученные результаты и получите общую площадь фигуры.


Также для вычисления площади прямоугольной трапеции подходят общие формулы для расчета площади трапеции.

  • Если известны длины оснований и высота (или перпендикулярная боковая сторона), то площадь рассчитывается по формуле:

S = (a + b) * h / 2

В качестве h (высоты) может выступать боковая сторона с. Тогда формула выглядит так:

S = (a + b) * c / 2

  • Другой способ рассчитать площадь — перемножить длину средней линии на высоту:

или на длину боковой перпендикулярной стороны:

  • Следующий способ вычисления — через половину произведения диагоналей и синус угла между ними:

S = ½ * d1 * d2 * sin α


Если диагонали перпендикулярны, то формула упрощается до:

S = ½ * d1 * d2

  • Еще один способ вычисления — через полупериметр (сумма длин двух противоположных сторон) и радиус вписанной окружности.

Эта формула действительна для оснований. Если брать длины боковых сторон, то одна из них будет равна удвоенному радиусу. Формула будет выглядеть так:

S = (2r + c) * r

  • Если в трапецию вписана окружность, то площадь вычисляется так же:

где m — длина средней линии.

Площадь криволинейной трапеции

Криволинейная трапеция представляет из себя плоскую фигуру, ограниченную графиком неотрицательной непрерывной функции y = f(x), определенной на отрезке , осью абсцисс и прямыми x = a, x = b. По сути, две ее стороны параллельны друг другу (основания), третья сторона перпендикулярна основаниям, а четвертая представляет из себя кривую, соответствующую графику функции.


Площадь криволинейной трапеции ищут через интеграл по формуле Ньютона-Лейбница:


Так вычисляются площади различных видов трапеций. Но, помимо свойств сторон, трапеции обладают одинаковыми свойствами углов. Как у всех существующих четырехугольников, сумма внутренних углов трапеции равна 360 градусов. А сумма углов, прилежащих к боковой стороне, — 180 градусам.

Практика прошлогодних ЕГЭ и ГИА показывает, что задачи по геометрии вызывают сложности у многих школьников. Вы легко справитесь с ними, если заучите все нужные формулы и попрактикуетесь в решении задач.

В этой статье вы увидите формулы нахождения площади трапеции, а также примеры задач с решениями. Такие же могут попасться вам в КИМах на аттестационных экзаменах или на олимпиадах. Поэтому отнеситесь к ним внимательно.

Что нужно знать про трапецию?

Для начала вспомним, что трапецией называется четырехугольник, у которого две противоположные стороны, их еще называют основаниями, параллельны, а две другие – нет.

В трапеции также может быть опущена высота (перпендикуляр к основанию). Проведена средняя линия – это прямая, которая параллельна основаниям и равна половине их суммы. А также диагонали, которые могут пересекаться, образуя острые и тупые углы. Или, в отдельных случаях, под прямым углом. Кроме того, если трапеция равнобедренная, в нее можно вписать окружность. И описать окружность около нее.

Формулы площади трапеции

Для начала рассмотрим стандартные формулы нахождения площади трапеции. Способы вычислить площадь равнобедренной и криволинейной трапеций рассмотрим ниже.

Итак, представьте, что у вас есть трапеция с основаниями a и b, в которой к большему основанию опущена высота h. Вычислить площадь фигуры в таком случае проще простого. Надо всего лишь разделить на два сумму длин оснований и умножить то, что получится, на высоту: S = 1/2(a + b)*h .

Возьмем другой случай: предположим, в трапеции, кроме высоты, проведена средняя линия m. Нам известна формула нахождения длины средней линии: m = 1/2(a + b). Поэтому с полным правом можем упростить формулу площади трапеции до следующего вида: S = m* h . Другими словами, чтобы найти площадь трапеции, надо умножить среднюю линию на высоту.

Рассмотрим еще один вариант: в трапеции проведены диагонали d 1 и d 2 , которые пересекаются не под прямым углом α. Чтобы вычислить площадь такой трапеции, вам нужно разделить на два произведение диагоналей и умножить то, что получится, на sin угла между ними: S= 1/2d 1 d 2 *sinα .

Теперь рассмотрим формулу для нахождения площади трапеции, если о ней неизвестно ничего, кроме длин всех ее сторон: a, b, c и d. Это громоздкая и сложная формула, но вам будет полезно запомнить на всякий случай и ее: S = 1/2(a + b) * √c 2 – ((1/2(b – a)) * ((b – a) 2 + c 2 – d 2)) 2 .

Кстати, приведенные выше примеры верны и для того случая, когда вам потребуется формула площади прямоугольной трапеции. Эта трапеция, боковая сторона которой примыкает к основаниям под прямым углом.

Равнобедренная трапеция

Трапеция, боковые стороны которой равны, называется равнобедренной. Мы рассмотрим несколько вариантов формулы площади равнобедренной трапеции.

Первый вариант: для случая, когда внутрь равнобедренной трапеции вписана окружность с радиусом r, а боковая сторона и большее основание образуют острый угол α. Окружность может быть вписана в трапецию при условии, что сумма длин ее оснований равна сумме длин боковых сторон.

Площадь равнобедренной трапеции вычисляется так: умножьте квадрат радиуса вписанной окружности на четыре и разделите все это на sinα: S = 4r 2 /sinα . Еще одна формула площади является частным случаем для того варианта, когда угол между большим основанием и боковой стороной равен 30 0: S = 8r 2 .

Второй вариант: на этот раз возьмем равнобедренную трапецию, в которой вдобавок проведены диагонали d 1 и d 2 , а также высота h. Если диагонали трапеции взаимно перпендикулярны, высота составляет половину суммы оснований: h = 1/2(a + b). Зная это, легко преобразовать уже знакомую вам формулу площади трапеции в такой вид: S = h 2 .

Формула площади криволинейной трапеции

Начнем с того, что разберемся: что такое криволинейная трапеция. Представьте себе ось координат и график непрерывной и неотрицательной функции f, которая не меняет знака в пределах заданного отрезка на оси x. Криволинейную трапецию образуют график функции у = f(x) – вверху, ось х – внизу (отрезок ), а по бокам – прямые, проведенные между точками a и b и графиком функции.

Вычислить площадь такой нестандартной фигуры нельзя приведенными выше способами. Тут нужно применить математический анализ и использовать интеграл. А именно: формулу Ньютона-Лейбница – S = ∫ b a f(x)dx = F(x)│ b a = F(b) – F(a) . В этой формуле F – первообразная нашей функции на выбранном отрезке . И площадь криволинейной трапеции соответствует приращению первообразной на заданном отрезке.

Примеры задач

Чтобы все эти формулы лучше улеглись в голове, вот вам несколько примеров задач на нахождение площади трапеции. Лучше всего будет, если вы сперва попробуете решить задачи сами, и только потом сверите полученный ответ с готовым решением.

Задача №1: Дана трапеция. Ее большее основание – 11 см, меньшее – 4см. В трапеции проведены диагонали, одна длиной 12 см, вторая – 9 см.

Решение: Постройте трапецию АМРС. Проведите прямую РХ через вершину Р так, чтобы она оказалась параллельной диагонали МС и пересекла прямую АС в точке Х. Получится треугольник АРХ.

Мы рассмотрим две полученных в результате этих манипуляций фигуры: треугольник АРХ и параллелограмм СМРХ.

Благодаря параллелограмму мы узнаем, что РХ = МС = 12 см и СХ = МР = 4см. Откуда можем вычислить сторону АХ треугольника АРХ: АХ = АС + СХ = 11 + 4 = 15 см.

Мы также можем доказать, что треугольник АРХ – прямоугольный (для этого примените теорему Пифагора – АХ 2 = АР 2 + РХ 2). И высчитать его площадь: S APX = 1/2(AP * PX) = 1/2(9 * 12) = 54 см 2 .

Дальше вам потребуется доказать, что треугольники АМР и РСХ являются равновеликими. Основанием послужит равенство сторон МР и СХ (уже доказанное выше). А также высоты, которые вы опустите на эти стороны – они равны высоте трапеции АМРС.

Все это позволит вам утверждать, что S AMPC = S APX = 54 см 2 .

Задача №2: Дана трапеция КРМС. На ее боковых сторонах расположены точки О и Е, при этом ОЕ и КС параллельны. Также известно, что площади трапеций ОРМЕ и ОКСЕ находятся в соотношении 1:5. РМ = а и КС = b. Требуется найти ОЕ.

Решение: Проведите через точку М прямую, параллельную РК, и точку ее пересечения с ОЕ обозначьте Т. А – точка пересечения прямой, проведенной через точку Е параллельно РК, с основанием КС.

Введем еще одно обозначение – ОЕ = х. А также высоту h 1 для треугольника ТМЕ и высоту h 2 для треугольника АЕС (вы можете самостоятельно доказать подобие этих треугольников).

Будем считать, что b > а. Площади трапеций ОРМЕ и ОКСЕ относятся как 1:5, что дает нам право составить такое уравнение: (х + а) * h 1 = 1/5(b + х) * h 2 . Преобразуем и получим: h 1 / h 2 = 1/5 * ((b + х)/(х + а)).

Раз треугольники ТМЕ и АЕС подобные, имеем h 1 / h 2 = (х – а)/(b – х). Объединим обе записи и получим: (х – а)/(b – х) = 1/5 * ((b + х)/(х + а)) ↔ 5(х – а)(х + а) = (b + х)(b – х) ↔ 5(х 2 – а 2) = (b 2 – х 2) ↔ 6х 2 = b 2 + 5а 2 ↔ х = √(5а 2 + b 2)/6.

Таким образом, ОЕ = х = √(5а 2 + b 2)/6.

Заключение

Геометрия не самая легкая из наук, но вы наверняка сможете справиться с экзаменационными заданиями. Достаточно проявить немного усидчивости при подготовке. И, конечно, запомнить все нужные формулы.

Мы постарались собрать в одном месте все формулы вычисления площади трапеции, чтобы вы могли воспользоваться ими, когда будете готовиться к экзаменам и повторять материал.

Обязательно расскажите про эту статью одноклассникам и друзьям в социальных сетях. Пускай хороших оценок за ЕГЭ и ГИА будет больше!

сайт, при полном или частичном копировании материала ссылка на первоисточник обязательна.

Трапецией называется четырехугольник, у которого только две стороны параллельны между собой.

Они называются основаниями фигуры, оставшиеся – боковыми сторонами. Частными случаями фигуры считается параллелограмм. Также существует криволинейная трапеция, которая включает в себя график функции. Формулы площади трапеции включают в себя практически все ее элементы, и лучшее решение подбирается в зависимости от заданных величин.
Основные роли в трапеции отводятся высоте и средней линии. Средняя линия – это линия, соединяющая середины боковых сторон. Высота трапеции проводится под прямым углом от верхнего угла к основанию.
Площадь трапеции через высоту равняется произведению полусуммы длин оснований, умноженному на высоту:

Если по условиям известна средняя линия, то эта формула значительно упрощается, так как она равна полусумме длин оснований :

Если по условиям даны длины всех сторон, то можно рассмотреть пример расчета площади трапеции через эти данные:

Допустим, дана трапеция с основаниями a = 3 см, b = 7 см и боковыми сторонами c = 5 см, d = 4 см. найдем площадь фигуры:

Площадь равнобокой трапеции


Отдельным случаем считается равнобокая или, как ее еще называют, равнобедренная трапеция.
Особым случаем является и нахождение площади равнобедренной (равнобокой) трапеции. Формула выводится различными способами – через диагонали, через углы, прилегающие к основанию и радиус вписанной окружности.
Если по условиям задана длина диагоналей и известен угол между ними можно использовать такую формулу:

Помните, что диагонали равнобокой трапеции равны между собой!


То есть, зная одно их оснований, сторону и угол, можно легко рассчитать площадь.

Площадь криволинейной трапеции


Отдельный случай – это криволинейная трапеция . Она располагается на оси координат и ограничивается графиком непрерывной положительной функции.

Ее основание располагает на оси X и ограничивается двумя точками:
Интегралы помогают вычислить площадь криволинейной трапеции.
Формула прописывается так:

Рассмотрим пример расчета площади криволинейной трапеции. Формула требует определенных знаний для работы с определенными интегралами. Для начала разберем значение определенного интеграла:

Здесь F(a) – это значение первообразной функции f(x) в точке a , F(b) – значение этой же функции f(x) в точке b .

Теперь решим задачу. На рисунке изображена криволинейная трапеция, ограниченная функцией . Функция
Нам необходимо найти площадь выделенной фигуры, которая является криволинейной трапецией, ограниченной сверху графиком , справа прямой x ={-8}, слева прямой x ={-10} и осью OX снизу.
Площадь этой фигуры мы будем рассчитывать по формуле:

Условиями задачи нам задана функция. По ней мы найдем значения первообразной в каждой из наших точек:

Теперь
Ответ: площадь заданной криволинейной трапеции равняется 4.

Ничего сложного в расчетах этого значения нет. Важна только предельная внимательность в вычислениях.

Для того чтобы чувствовать себя на уроках геометрии уверенно и успешно решать задачи, недостаточно выучить формулы. Их нужно в первую очередь понимать. Бояться, а тем более ненавидеть формулы — непродуктивно. В этой статье доступным языком будут проанализированы различные способы поиска площади трапеции. Для лучшего усвоения соответствующих правил и теорем уделим некоторое внимание ее свойствам. Это поможет разобраться в том, как работают правила и в каких случаях следует применять те или иные формулы.

Определяем трапецию

Что это за фигура в целом? Трапецией называют многоугольник из четырех углов с двумя параллельными сторонами. Две другие стороны трапеции могут быть наклонены под различными углами. Ее параллельные стороны называют основаниями, а для непараллельных сторон применяют наименование «боковые стороны» или «бедра». Такие фигуры довольно часто встречаются в обыденной жизни. Контуры трапеции можно увидеть в силуэтах одежды, предметах интерьера, мебели, посуды и многих других. Трапеция бывает разных видов: разносторонняя, равнобокая и прямоугольная. Более детально их типы и свойства разберем далее в статье.

Свойства трапеции

Остановимся коротко на свойствах этой фигуры. Сумма углов, прилегающих к любой боковой стороне, всегда равняется 180°. Надо заметить, что все углы трапеции в сумме составляют 360°. У трапеции существует понятие средней линии. Если соединить середины боковых сторон отрезком — это и будет средняя линия. Ее обозначают m. У средней линии есть важные свойства: она всегда параллельна основаниям (мы помним, что основания также параллельны между собой) и равна их полусумме:

Это определение обязательно надо выучить и понять, ведь это ключ к решению множества задач!

У трапеции всегда можно опустить высоту на основание. Высота — это перпендикуляр, часто обозначаемый символом h, который проведен из любой точки одного основания на другое основание или его продолжение. Средняя линия и высота помогут найти площадь трапеции. Подобные задачи являются самыми распространенными в школьном курсе геометрии и регулярно появляются среди контрольных и экзаменационных работ.

Самые простые формулы площади трапеции

Разберем две самые популярные и простые формулы, с помощью которых находят площадь трапеции. Достаточно умножить высоту на полусумму оснований, чтобы легко найти искомое:

S = h*(a + b)/2.

В этой формуле a, b обозначают основания трапеции, h — высоту. Для удобства восприятия в этой статье знаки умножения отмечены символом (*) в формулах, хотя в официальных справочниках знак умножения обычно опускают.

Рассмотрим пример.

Дано: трапеция с двумя основаниями, равными 10 и 14 см, высота составляет 7 см. Чему равна площадь трапеции?

Разберем решение этой задачи. По этой формуле сначала нужно найти полусумму оснований: (10+14)/2 = 12. Итак, полусумма равняется 12 см. Теперь полусумму умножаем на высоту: 12*7 = 84. Искомое найдено. Ответ: площадь трапеции равна 84 кв. см.

Вторая известная формула гласит: площадь трапеции равна произведению средней линии на высоту трапеции. То есть фактически вытекает из предшествующего понятия средней линии: S=m*h.

Использование диагоналей для вычислений

Другой способ нахождения площади трапеции на самом деле не так уж сложен. Он связан с ее диагоналями. По этой формуле для нахождения площади требуется умножить полупроизведение ее диагоналей (d 1 d 2) на синус угла между ними:

S = ½ d 1 d 2 sina.

Рассмотрим задачу, которая показывает применение этого способа. Дано: трапеция с длиной диагоналей равной соответственно 8 и 13 см. Угол a между диагоналями равняется 30°. Найти площадь трапеции.

Решение. Используя вышеприведенную формулу, легко вычислить требуемое. Как известно, sin 30° составляет 0,5. Следовательно, S = 8*13*0,5=52. Ответ: площадь равна 52 кв. см.

Ищем площадь равнобокой трапеции

Трапеция может быть равнобокой (равнобедренной). Ее боковые стороны одинаковы И углы при основаниях равны, что хорошо иллюстрирует рисунок. Равнобедренная трапеция имеет такие же свойства, что и обычная, плюс ряд особых. Вокруг равнобокой трапеции может быть описана окружность, и в нее может быть вписана окружность.

Какие же есть методики вычисления площади такой фигуры? Нижеприведенный способ потребует больших вычислений. Для его применения нужно знать значения синуса (sin) и косинуса (cos) угла при основании трапеции. Для их расчетов требуются либо таблицы Брадиса либо инженерный калькулятор. Вот эта формула:

S = c *sin a *(a c *cos a ),

где с — боковое бедро, a — угол при нижнем основании.

Равнобокая трапеция обладает диагоналями одинаковой длины. Верно и обратное утверждение: если у трапеции диагонали равны, то она является равнобедренной. Отсюда следующая формула, помогающая найти площадь трапеции — полупроизведение квадрата диагоналей на синус угла между ними: S = ½ d 2 sina.

Находим площадь прямоугольной трапеции

Известен частный случай прямоугольной трапеции. Это трапеция, у которой одна боковая сторона (ее бедро) примыкает к основаниям под прямым углом. Она имеет свойства обычной трапеции. Помимо этого, она обладает очень интересной особенностью. Разность квадратов диагоналей такой трапеции равняется разности квадратов ее оснований. Для нее используют все ранее приведенные методики вычисления площади.

Применяем смекалку

Есть одна хитрость, которая может помочь в случае забывчивости специфических формул. Рассмотрим внимательнее, что представляет собой трапеция. Если мысленно разделить ее на части, то мы получим знакомые и понятные геометрические фигуры: квадрат или прямоугольник и треугольник (один или два). Если известны высота и стороны трапеции, можно воспользоваться формулами площади треугольника и прямоугольника, после чего сложить все полученные величины.

Проиллюстрируем это следующим примером. Дана прямоугольная трапеция. Угол C = 45°, углы A, D составляют 90°. Верхнее основание трапеции равно 20 см, высота равна 16 см. Требуется вычислить площадь фигуры.

Данная фигура очевидным образом состоит из прямоугольника (если два угла равны 90°) и треугольника. Так как трапеция прямоугольная, следовательно, ее высота равна ее боковой стороне, то есть 16 см. Имеем прямоугольник со сторонами 20 и 16 см соответственно. Рассмотрим теперь треугольник, угол которого равен 45°. Мы знаем, что одна его сторона составляет 16 см. Так как эта сторона является одновременно высотой трапеции (а нам известно, что высота опускается на основание под прямым углом), следовательно, второй угол треугольника равен 90°. Отсюда оставшийся угол треугольника составляет 45°. Следствием этого мы получаем прямоугольный равнобедренный треугольник, у которого две стороны одинаковы. Значит, другая сторона треугольника равна высоте, то есть 16 см. Осталось вычислить площадь треугольника и прямоугольника и сложить полученные величины.

Площадь прямоугольного треугольника равна половине произведения его катетов: S = (16*16)/2 = 128. Площадь прямоугольника равняется произведению его ширины на длину: S = 20*16 = 320. Мы нашли требуемое: площадь трапеции S = 128 + 320 = 448 кв. см. Можно легко себя перепроверить, воспользовавшись вышеприведенными формулами, ответ будет идентичен.

Используем формулу Пика


Напоследок приведем еще одну оригинальную формулу, помогающую искать площадь трапеции. Она называется формулой Пика. Ею удобно пользоваться, когда трапеция нарисована на клетчатой бумаге. Подобные задачи часто встречаются в материалах ГИА. Выглядит она следующим образом:

S = M/2 + N — 1,

в этой формуле M — количество узлов, т. е. пересечений линий фигуры с линиями клетки на границах трапеции (оранжевые точки на рисунке), N — количество узлов внутри фигуры (синие точки). Удобнее всего пользоваться ею при нахождении площади неправильного многоугольника. Тем не менее, чем больше арсенал используемых методик, тем меньше ошибок и лучше результаты.

Разумеется, приведенными сведениями далеко не исчерпываются типы и свойства трапеции, а также способы поиска ее площади. В этой статье дан обзор наиболее важных ее характеристик. В решении геометрических задач важно действовать постепенно, начинать с легких формул и задач, последовательно закреплять понимание, переходить на другой уровень сложности.

Собранные воедино самые распространенные формулы помогут ученикам сориентироваться в разнообразных способах вычисления площади трапеции и более качественно подготовиться к тестам и контрольным работам по этой теме.

Существует множество способов найти площадь трапеции. Обычно репетитор по математике владеет несколькими приемами ее вычисления, остановимся на них подробнее:
1) , где AD и BC основания, а BH-высота трапеции. Доказательство: проведем диагональ BD и выразим площади треугольников ABD и CDB через полупроизведение их оснований на высоту:

, где DP – внешняя высота в

Сложим почленно эти равенства и учитывая, что высоты BH и DP равны, получим:

Вынесем за скобку

Что и требовалось доказать.

Следствие из формулы площади трапеции:
Так как полусумма оснований равна MN — средней линии трапеции, то

2) Применение общей формулы площади четырехугольника .
Площадь четырехугольника равна половине произведения диагоналей, умноженной на синус угла между ними
Для доказательства достаточно разбить трапецию на 4 треугольника, выразить площадь каждого через «половину произведения диагоналей на синус угла между ними» (в качестве угла берется , сложить получившиеся выражения, вынести за скобку и раскладываю эту скобку на множители методом группировки получить ее равенство выражению . Отсюда

3) Метод сдвига диагонали
Это мое название. В школьных учебниках репетитор по математике не встретит такого заголовка. Описание приема можно найти только в дополнительных учебных пособиях в качестве примера решения какой-нибудь задачи. Отмечу, что большинство интересных и полезных фактов планиметрии репетиторы по математике открывают ученикам в процессе выполнения практической работы. Это крайне неоптимально, ибо школьнику нужно выделять их в отдельные теоремы и называть «громкими именами». Одно из таких – «сдвиг диагонали». О чем идет речь? Проведем через вершину B прямую параллельную к АС до пересечения с нижним основанием в точке E. В таком случае четырехугольник EBCA будет параллелограммом (по определению) и поэтому BC=EA и EB=AC. Нам сейчас важно первое равенство. Имеем:

Заметим, что треугольник BED, площадь которого равна площади трапеции, имеет еще несколько замечательных свойств:
1) Его площадь равна площади трапеции
2) Его равнобедренность происходит одновременно с равнобедренность самой трапеции
3) Верхний его угол при вершине B равен углу между диагоналями трапеции (что очень часто используется в задачах)
4) Его медиана BK равна расстоянию QS между серединами оснований трапеции. С применением этого свойства я недавно столкнулся при подготовке ученика на мехмат МГУ по учебнику Ткачука, вариант 1973 года (задача приводится внизу страницы).

Спецприемы репетитора по математике.

Иногда я предлагаю задачи на весьма хитрый путь нахождении я площади трапеции. Я отношу его к спецприемам ибо на практике репетитор их использует крайне редко. Если вам нужна подготовка к ЕГЭ по математике только в части B, можно про них и не читать. Для остальных рассказываю дальше. Оказывается площадь трапеции в два раза больше площади треугольника с вершинами в концах одной боковой стороны и серединой другой, то есть треугольника ABS на рисунке:
Доказательство: проведем высоты SM и SN в треугольниках BCS и ADS и выразим сумму площадей этих треугольников:

Так как точка S – середина CD, то (докажите это сами).Найдем cумму площадей треугольников:

Так как эта сумма оказалась равной половине площади трапеции, то — вторая ее половина. Ч.т.д.

В копилку спецприемов репетитора я бы отнес форму вычисления площади равнобедренной трапеции по ее сторонам: где p – полупериметр трапеции. Доказательство я приводить не буду. Иначе ваш репетитор по математике останется без работы:). Приходите на занятия!

Задачи на площадь трапеции:

Замечание репетитора по математике : Нижеприведенный список не является методическим сопровождением к теме, это только небольшая подборка интересных задач на вышерассмотренные приемы.

1) Нижнее основание равнобедренной трапеции равно 13, а верхнее равно 5. Найдите площадь трапеции, если ее диагональ перпендикулярна боковой стороне.
2) Найдите площадь трапеции, если ее основания равны 2см и 5см, а боковые стороны 2см и 3см.
3) В равнобокой трапеции большее основание равно 11, боковая сторона равна 5, а диагональ равна Найти площадь трапеции.
4) Диагональ равнобокой трапеции равна 5, а средняя линия равна 4. Найти площадь.
5) В равнобедренной трапеции основания равны 12 и 20, а диагонали взаимно перпендикулярны. Вычислить площадь трапеции
6) Диагональ равнобокой трапеции составляет с ее нижним основанием угол . Найти площадь трапеции, если ее высота равна 6см.
7) Площадь трапеции равна 20, а одна из ее боковых сторон равна 4 см. Найдите расстояние до нее от середины противоположной боковой стороны.
8) Диагональ равнобокой трапеции делит ее на треугольники с площадями 6 и 14. Найти высоту, если боковая сторона равна 4.
9) В трапеции диагонали равны 3 и 5, а отрезок, соединяющий середины оснований равен 2. Найти площадь трапеции (Мехмат МГУ, 1970г).

Я выбирал не самые сложные задачи (не стоит пугаться мехмата!) с расчетом на возможность их самостоятельного решения. Решайте на здоровье! Если вам нужна подготовка к ЕГЭ по математике, то без участия в этом процессе формулы площади трапеции могут возникнуть серьезные проблемы даже с задачей B6 и тем более с C4. Не запускайте тему и в случае каких-либо затруднений обращайтесь за помощью. Репетитор по математике всегда рад вам помочь.

Колпаков А.Н.
Репетитор по математике в Москве , подготовка к ЕГЭ в Строгино .

 

Как найти площадь трапеции

Если вы считаете, что контент, доступный через Веб-сайт (как определено в наших Условиях обслуживания), нарушает одно или больше ваших авторских прав, сообщите нам, отправив письменное уведомление («Уведомление о нарушении»), содержащее в информацию, описанную ниже, назначенному ниже агенту. Если репетиторы университета предпримут действия в ответ на ан Уведомление о нарушении, оно предпримет добросовестную попытку связаться со стороной, которая предоставила такой контент средствами самого последнего адреса электронной почты, если таковой имеется, предоставленного такой стороной Varsity Tutors.

Ваше Уведомление о нарушении прав может быть отправлено стороне, предоставившей доступ к контенту, или третьим лицам, таким как в виде ChillingEffects. org.

Обратите внимание, что вы будете нести ответственность за ущерб (включая расходы и гонорары адвокатам), если вы существенно искажать информацию о том, что продукт или действие нарушает ваши авторские права. Таким образом, если вы не уверены, что контент находится на Веб-сайте или по ссылке с него нарушает ваши авторские права, вам следует сначала обратиться к юристу.

Чтобы отправить уведомление, выполните следующие действия:

Вы должны включить следующее:

Физическая или электронная подпись правообладателя или лица, уполномоченного действовать от их имени; Идентификация авторских прав, которые, как утверждается, были нарушены; Описание характера и точного местонахождения контента, который, по вашему мнению, нарушает ваши авторские права, в \ достаточно подробностей, чтобы позволить репетиторам Varsity найти и точно идентифицировать этот контент; например нам требуется а ссылка на конкретный вопрос (а не только на название вопроса), который содержит содержание и описание к какой конкретной части вопроса — изображению, ссылке, тексту и т. д. — относится ваша жалоба; Ваше имя, адрес, номер телефона и адрес электронной почты; и Ваше заявление: (а) вы добросовестно считаете, что использование контента, который, по вашему утверждению, нарушает ваши авторские права не разрешены законом, владельцем авторских прав или его агентом; (б) что все информация, содержащаяся в вашем Уведомлении о нарушении, является точной, и (c) под страхом наказания за лжесвидетельство вы либо владелец авторских прав, либо лицо, уполномоченное действовать от их имени.

Отправьте жалобу нашему уполномоченному агенту по адресу:

Чарльз Кон Varsity Tutors LLC
101 S. Hanley Rd, Suite 300
St. Louis, MO 63105

Или заполните форму ниже:

Как найти площадь трапеции (формула и видео) // Tutors.com

Содержание

  1. Что такое трапеция?
  2. Как найти площадь трапеции
  3. Площадь трапеции, формула
  • Площадь трапеции Примеры
  • Трапеция представляет собой четырехугольник с одной парой параллельных сторон. Итак, этот четырехсторонний многоугольник представляет собой плоскую фигуру и замкнутую фигуру. Он состоит из четырех отрезков и четырех внутренних углов. Параллельные стороны — это два основания трапеции; две другие стороны — его ноги.

    Обычно у трапеции более длинная параллельная сторона — основание , — горизонтально. Перпендикулярная линия от основания к другой параллельной стороне даст вам высоту трапеции или высоту .

    Что такое средний по математике?

    В математике среднее значение — это сумма группы чисел, деленная на количество элементов в группе.

    Итак, если у вас есть три человека, которые держат книги, вы можете найти среднее количество книг, которые они держат, вот так: Мартин держит 5 книг, Мак держит 3 книги, а Мария держит 4 книги. Вместе 12 книг держат 3 человека. Итак, 12 книг ÷ 3 человека = в среднем по 4 книги.

    Чтобы найти площадь трапеции, вы найдете среднюю длину двух оснований.

    Как найти площадь трапеции

    Чтобы найти площадь любой трапеции, начните с обозначения ее основания и высоты.На нашей трапеции обозначьте более длинное основание a и более короткое основание b. Обозначьте линию, перпендикулярную двум основаниям, h для высоты или высоты трапеции.

    Обратите внимание, мы не пометили ноги. Нам не нужно ничего знать о длине ног или углах вершин, чтобы найти площадь.

    Площадь трапеции, формула

    Формула площади трапеции — это среднее значение оснований, умноженное на высоту. В формуле длинное и короткое основание — это a и b, а высота — h:

    .

    Умножение на 12 аналогично делению на 2.Мы берем половину суммы длины двух оснований (их среднее значение), а затем умножаем это на высоту или высоту, чтобы найти площадь в квадратных единицах.

    Уравнение площади трапеции

    Трапеция LMNO имеет параллельные основания LM и NO. Линейный сегмент LM имеет длину 7 см, а линейный сегмент NO — 13 см. Мы обозначим более длинную сторону NO как a, а короткую сторону LM как b. Высота h 5 см.

    Сначала давайте подставим эти числа в нашу формулу:

    площадь = 13 см + 7 см2 × 5 см

    Далее складываем 13 плюс 7 и получаем:

    площадь = 20 см2 × 5 см

    Потом делим на два и получаем:

    площадь = 10 см × 5 см

    Наконец, умножаем и получаем ответ:

    площадь = 50 см2

    Площадь этой трапеции составляет 50 квадратных сантиметров.

    Площадь трапеции Примеры

    Теперь попробуйте! Другая трапеция имеет длинное основание a, 11 метров, и более короткое основание b, 7 метров. Его высота h составляет 9 метров. Какая площадь в квадратных метрах?

    площадь = 11 см + 7 см2 × 9 см

    Получили 81 квадратный метр? Ваш ответ для площади всегда выражается в квадратных единицах линейного измерения. Таким образом, трапеция, измеренная в футах, дает площадь в квадратных футах, сантиметры — в квадратных сантиметрах и так далее.

    Помните, что умножение на 1/2 аналогично делению на 2, поэтому вы можете сложить длины оснований, а затем разделить их сумму на два, если вам так легче.

    Из-за коммутативности умножения вы можете переставить эти три числа, 12, высоту h и длину основания a + b, в любом порядке, чтобы упростить вычисления.

    Итак, с трапецией LMNO вы могли бы написать такую ​​формулу, как:

    площадь = 12 × 9 × (11 + 7)

    Пример # 2

    Вот вам еще один пример. Новая трапеция перевернута по сравнению с тем, как вы их обычно видите, но пусть это вас не остановит! Короткое основание b имеет длину 21 дюйм.Длинное основание a (на этот раз вверху рисунка) составляет 31 дюйм в длину. Высота h (независимо от того, с какой стороны вы смотрите на трапецию) составляет 5 дюймов.

    площадь = 12 × 5 × (31 + 21)

    ИЛИ

    площадь = 12 × (31 + 21) × 5

    ИЛИ

    площадь = 31 + 212 × 5

    Как бы вы ни использовали формулу, вы всегда получите один и тот же ответ: площадь = 130 кв. Дюймов

    Краткое содержание урока

    В этом уроке и видео мы рассмотрели, что такое трапеция, изучили, как средние значения играют роль в геометрии, узнали, как маркировать и использовать части трапеции для вычисления площади, а также узнали формулу для вычисления площади трапеции в квадратные единицы.

    Следующий урок:

    Формула Герона

    Калькулятор площади трапеции [Простота использования + руководство по результатам]

    Как рассчитать площадь трапеции

    Трапеция — это интересная четырехгранная геометрическая фигура. У него две параллельные стороны, а оставшиеся две стороны могут быть любой длины и под любым углом. Некоторые возможные формы трапеции показаны ниже для пояснения концепции. Обратите внимание, что параллельные линии отмечены стрелками.

    В реальной жизни существует множество объектов трапециевидной формы, которые вы могли или не могли заметить. См. Несколько примеров ниже. Вы удивлены?

    Теперь, когда вы знаете форму трапеции, давайте обсудим параметры, которые вам нужно знать, чтобы определить ее площадь. Есть три важных значения длины, которые вам нужно знать, чтобы определить площадь трапеции: длины двух параллельных сторон «a» и «b» и высота. Высота — это расстояние по перпендикуляру между двумя параллельными сторонами.Под перпендикулярным расстоянием мы подразумеваем, что длина линии, соединяющей параллельные стороны «a» и «b», составляет ровно 90 градусов к ним.

    Площадь трапеции, A , определяется как:

    Эта формула выводится из концепции площади треугольника. Возможно, вы уже знаете, как рассчитать площадь треугольника, но мы кратко рассмотрим это на случай, если вы забыли или не знаете. Два параметра, которые вам нужно знать, чтобы найти площадь треугольника, — это высота треугольника и основание треугольника. Высота треугольника определяется как расстояние по перпендикуляру от одного угла треугольника до уровня основания. Какую бы сторону треугольника вы ни выбрали в качестве «основания», измерьте высоту, считая угол точно противоположным основанию. См. Диаграммы ниже для большей ясности в отношении концепции высоты основания.

    Не запутайтесь, если форма треугольника не соответствует вашим ожиданиям. Помните о концепции основания, высоты и надписи соответственно.

    Площадь треугольника определяется как:

    Итак, как эти знания помогают нам в вычислении формулы площади трапеции? Посмотрим.

    Посмотрите внимательно, и вы заметите, что трапецию можно разрезать по диагонали, чтобы образовать два треугольника:

    Если мы найдем площадь этих двух треугольников и затем сложим их, мы получим площадь всей трапеции! Основание верхнего треугольника имеет длину «а», а основание нижнего треугольника — длину «b». Высота обоих треугольников одинакова.

    Площадь верхнего треугольника определяется как:

    Площадь нижнего треугольника определяется как:

    Следовательно, площадь трапеции будет:

    Принимая \ (\ frac {h } {2} \) в качестве общего множителя получаем:

    Надеюсь, теперь вы полностью понимаете концепцию формулы площади трапеции.Приведем несколько примеров.

    Пример 1:

    Найдите площадь трапеции, указанную ниже:

    Решение:

    Из рисунка видно, что:

    a = 4 см

    b = 9 см

    h = 5 см

    Пусть площадь трапеции представлена ​​переменной ‘A’

    A =?

    Примените формулу для площади трапеции:

    Пример 2:

    Трапеция площадью 98 см 2 имеет две параллельные стороны длиной 16 см и 12 см.Каково перпендикулярное расстояние между двумя параллельными сторонами?

    Решение:

    Нам даны следующие параметры:

    Параллельная сторона 1 = a = 16 см

    Параллельная сторона 2 = b = 12 см

    Площадь трапеции = A = 98 см 2

    Нам нужно найти расстояние по перпендикуляру между двумя параллельными сторонами. Как мы уже упоминали ранее в статье, это высота трапеции.

    ч =?

    Вспомните формулу площади трапеции и решите относительно «h».

    Теперь мы вводим известные значения и находим высоту:

    Пример 3:

    Площадь трапеции, представленной ниже, составляет 100 см 2 . Найдите неизвестную длину параллельной стороны «a».

    Решение:

    Одна сторона этой трапеции образует угол 90 градусов с обеими параллельными сторонами. Это означает, что высота трапеции и длина этой стороны одинаковы.Здесь приводится следующая информация:

    Площадь трапеции = A = 100 см 2

    Высота = h = 10 см

    Параллельная сторона 2 = b = 11 см

    Параллельная сторона 1 = a =?

    Чтобы найти «а», мы изменим формулу площади трапеции, чтобы найти «а»:

    Теперь введите известные значения, чтобы получить окончательный ответ:

    Заключительные мысли!

    Мы постарались охватить практически все, что нужно знать о площади трапеции, от ее образования до решения различных проблем. Геометрия — очень важный раздел математики, и изучение всех форм, существующих в реальном мире, имеет решающее значение, особенно если вы думаете однажды стать инженером! Изучив теорию, вы можете использовать нашу область калькулятора трапеций, чтобы быстро получить ответы на свои проблемы и сэкономить время!

    Объяснитель урока: Площадь трапеции

    В этом объяснении мы узнаем, как найти площадь трапеции с помощью формулы и применить ее для определения площади в реальной жизни.

    Мы начнем с обсуждения различных типов трапеций, которые существуют. Трапеция — это четырехугольник, у которого ровно одна пара параллельных сторон. Мы называем эти стороны основаниями и часто обозначаем их длину буквами и 𝑏. Перпендикулярное расстояние между двумя основаниями называется высотой трапеции и обычно обозначается ℎ. Две другие стороны трапеции (непараллельные противоположные стороны) известны как ноги.

    Равнобедренная трапеция — это трапеция, у которой ноги имеют одинаковую длину. У всех равнобедренных трапеций есть линия симметрии, проходящая через середины их оснований.

    A правая трапеция — это трапеция, в которой одна из опор перпендикулярна двум параллельным основаниям.

    Теперь мы рассмотрим, как найти площадь трапеции, в которой мы обозначим высоту через и длину основаниями и.

    Предположим, мы рисуем диагональ трапеции, соединяющую две противоположные вершины. Это разделит трапецию на два треугольника, как показано на рисунке ниже.

    Площадь каждого треугольника можно найти по формуле площадь перпендикулярной высоты треугольника = × 2.

    Высота перпендикуляра каждого треугольника равна ℎ. Верхний треугольник имеет основание длиной единиц, а нижний треугольник имеет основание. база длиной единицы. Следовательно, площадь трапеции определяется выражением площадь рапезоида, площадь верхнего треугольника, площадь нижнего треугольника = + = 𝑎 × ℎ2 + 𝑏 × ℎ2 = (𝑎 + 𝑏) ℎ2.

    Отметим, что (𝑎 + 𝑏) — это сумма длин параллельных оснований трапеции. Неформально мы можем думать о площадь трапеции как «половину суммы параллельных оснований, умноженной на высоту».

    Формула: Площадь трапеции

    Площадь трапеции равна половине суммы длин параллельных оснований, умноженной на высоту.

    Для трапеции высотой и основанием длиной 𝑎 и площадь определяется по формуле areaoftrapezoid = 12 (𝑎 + 𝑏) ℎ.

    В нашем первом примере мы продемонстрируем, как применить эту формулу, чтобы найти площадь трапеции с учетом ее высоты и длины ее оснований.

    Пример 1: Определение площади трапеции

    Параллельные стороны трапеции имеют длину 82 и 70. Если высота равна 100, какова площадь трапеции?

    Ответ

    Напомним, что площадь трапеции определяется как areaoftrapezoid = 12 (𝑎 + 𝑏) ℎ, где 𝑎 и 𝑏 обозначают длины оснований или параллельных сторон трапеции. и ℎ представляет его перпендикулярную высоту. Нам дана каждая из этих длин в вопросе.

    Подставляя 𝑎 = 82, 𝑏 = 70 и ℎ = 100, а затем вычисляя, получаем areaoftrapezoid = 12 (82 + 70) × 100 = 12 × 152 × 100 = 76 × 100 = 7600.

    Следовательно, площадь трапеции составляет 7 600 квадратных единиц.

    Мы рассмотрели пример вычисления площади трапеции с учетом ее высоты и длины параллельных сторон. Теперь рассмотрим проблему это по сути обратное: вычисление высоты трапеции с учетом ее площади и длины ее оснований.

    Пример 2: Определение высоты трапеции с учетом ее площади

    Площадь этой трапеции составляет 30 000 ярдов 2 . Какая у него высота?

    Ответ

    Начнем с того, что вспомним, что площадь трапеции можно вычислить, умножив половину суммы длин параллельных оснований на высоту перпендикуляра.

    Из рисунка мы видим, что параллельные основания трапеции имеют длину 80 ярдов и 295 ярдов. Нам также дана длина одной ножки трапеции. (232 ярда), но это не имеет отношения к нашему расчету, так как это не перпендикулярная высота трапеции.

    Мы можем использовать заданную площадь трапеции и длины двух параллельных оснований, чтобы сформировать уравнение, где ℎ представляет собой неизвестную высоту трапеции: 12 (80 + 295) ℎ = 30000.

    Теперь решим это уравнение, чтобы определить значение. Упрощение левой части дает 3752ℎ = 30000.

    Умножение обеих частей уравнения на 2375 (обратное умножение 3752) дает ℎ = 30000 × 2375 = 60000375 = 160.

    Следовательно, высота трапеции составляет 160 ярдов.

    В примере, который мы только что видели, нам было предоставлено больше информации, чем нам нужно на рисунке. Длина ножки трапеции не требовалась для того, чтобы рассчитать его площадь. Понимание измерений, необходимых для применения определенной формулы, и умение выбирать соответствующую информацию из диаграммы или формулировка описания — важные навыки при решении геометрических задач.

    В нашем следующем примере мы рассмотрим, как найти длину одной из параллельных сторон трапеции с учетом ее площади, высоты, и длина другой параллельной стороны.

    Пример 3: Определение длины основания трапеции с учетом ее площади

    Трапеция имеет площадь 1 760, а расстояние между ее параллельными сторонами равно 40. Если одна параллельная сторона равна 39, какова другая сторона?

    Ответ

    Напомним, что площадь трапеции с параллельными сторонами (или основаниями) длиной и единиц и высота ℎ единиц определяется выражением areaoftrapezoid = 12 (𝑎 + 𝑏) ℎ.

    Нам дано, что эта трапеция имеет площадь 1 760 квадратных единиц.Расстояние между параллельными сторонами — это еще один способ обозначить высоту. трапеции составляет 40 единиц, а длина одной параллельной стороны или основания трапеции составляет 39 единиц. Подставляя каждое из этих значений в формулу Выше приведено уравнение, которое мы можем решить, чтобы определить длину другой параллельной стороны: 1760 = 12 (39 + 𝑏) × 40.

    Начнем с упрощения правой части уравнения, отбрасывая множитель 2: 1760 = 20 (39 + 𝑏).

    Разделив обе части уравнения на 20, получим 88 = 39 + 𝑏.

    Наконец, вычитание 39 из каждой части уравнения дает 49 = 𝑏.

    Следовательно, длина другой параллельной стороны (или основания) трапеции составляет 49 единиц.

    Теперь рассмотрим альтернативный способ задания формулы площади трапеции. Неформально мы сказали, что эту формулу можно представить как «Половина суммы параллельных оснований, умноженная на высоту». Фактически, «половина суммы параллельных оснований» имеет геометрическое значение, которое мы определим ниже.

    Определение: Среднее основание трапеции

    Среднее основание трапеции — это отрезок прямой, концы которого являются серединами сторон трапеции. Среднее основание трапеции параллельна двум основаниям трапеции.

    Длина среднего основания трапеции, 𝑚, является средним арифметическим длин оснований и 𝑏: 𝑚 = 𝑎 + 𝑏2.

    Другими словами, длина среднего основания трапеции равна «половине суммы длин параллельных оснований.”Формула для площадь трапеции может быть эквивалентно выражена как площадь рапезоида, длина среднего основания, высота = ×.

    Формула: Площадь трапеции с использованием длины ее среднего основания

    Площадь трапеции равна длине ее среднего основания, умноженной на ее высоту.

    Для трапеции высотой и среднего основания длиной площадь определяется как areaoftrapezoid = 𝑚ℎ.

    Теперь мы рассмотрим два примера, в которых мы используем эту версию формулы площади для решения двух задач, связанных с площадью трапеции.

    Пример 4: Использование среднего основания для определения площади трапеции

    Найдите площадь показанной трапеции.

    Ответ

    При осмотре схемы мы замечаем, что 𝑋𝑌 разделяет каждую из сторон трапеции, 𝐴𝐷 и 𝐵𝐶 на два отрезка равной длины. Следовательно, 𝑋𝑌 соединяет середины ножек трапеции и, следовательно, является средним основанием трапеции.

    Напомним, что площадь трапеции можно рассчитать исходя из длины ее среднего основания и высоты по формуле площадь рапезоида, длина среднего основания, высота = ×.

    Заменяя длину среднего основания 19 мм. и 8 мм на высоту трапеции дает areaoftrapezoidmm = 19 × 8 = 152.

    Пример 5: Расчет средней длины основания трапеции с учетом ее площади и высоты

    Найдите среднюю длину основания трапеции с площадью 28 см 2 и высотой 4 см.

    Ответ

    Напомним, что площадь трапеции можно рассчитать по формуле площадь рапезоида, длина среднего основания, высота = ×.

    Нам даны площадь и высота трапеции, и мы можем составить уравнение. Подставив 28 вместо площади и 4 вместо высоты, мы получим 28 = × 4.lengthofmiddlebase

    Чтобы найти длину среднего основания, разделим обе части уравнения на 4: lengthofmiddlebase = 284 = 7.

    Длина среднего основания данной трапеции составляет 7 см.

    Методы, которые мы разработали в этом объяснителе, также могут быть применены к реальным задачам, связанным с трапециями.Теперь рассмотрим последний пример. связаны с размерами фермерских полей, одно из которых имеет форму трапеции.

    Пример 6: Использование площадей трапеций для решения реальной проблемы

    Фермер владеет двумя полями равной площади: одно в форме ромба, а другое в форме трапеции, как показано на рисунке. Рассчитайте длину среднего основания трапециевидного поля.

    Ответ

    Начнем с рассмотрения поля слева на рисунке.Это поле имеет форму ромба, потому что все его четыре стороны равны по длине. Нам даны длины двух диагоналей ромба: они равны 100 м и 90 м. Затем напомним, что площадь ромба определяется как areaofrhombus = 𝑑𝑑2,  где 𝑑 и 𝑑 обозначают длины его диагоналей. Следовательно, мы имеем areaofrhombusm = 100 × 902 =

    = 4500.

    Далее мы рассматриваем поле справа на рисунке. Это поле имеет форму трапеции, потому что это четырехугольник с одной парой параллельных сторон.Нам дана высота трапеции (25 м), и теперь мы знаем, что ее площадь, которая совпадает с площадью другого месторождения — 4 500 м 2 .

    Поскольку мы хотим рассчитать длину среднего основания трапеции, мы напоминаем формулу для площади трапеции, которая включает эту меру: площадь рапезоида, длина среднего основания, высота = ×.

    Замена известной площади трапеции (4500 м 2 ) на известную высоту (25 м) дает уравнение, которое мы можем решить для определения длины среднего основания: 4500 = × 25.lengthofmiddlebase

    Если разделить обе части этого уравнения на 25, получим lengthofmiddlebasem = 450025 = 180.

    Длина средней базы поля в форме трапеции — 180 м.

    В заключение напомним некоторые ключевые моменты.

    Ключевые точки

    • Площадь трапеции с высотой ℎ и параллельными основаниями длиной 𝑎 и определяется выражением areaoftrapezoid = 12 (𝑎 + 𝑏) ℎ.
    • Мы можем неформально думать об этом как площадь рапезоида равна половине высоты параллельного основания = ×.
    • Среднее основание трапеции — это отрезок прямой, концы которого являются серединами двух сторон трапеции.
    • Длина среднего основания трапеции, 𝑚, является средним арифметическим длин оснований: 𝑚 = 𝑎 + 𝑏2.
    • Площадь трапеции равна длине ее среднего основания, умноженной на ее высоту: площадь рапезоида, длина среднего основания, высота = ×.

    Измерение площадей (предварительная алгебра, площадь и объем) — Mathplanet

    Площадь параллелограмма измеряется так же, как площадь прямоугольника или квадрата.{2} $$


    Как мы показали в предыдущем разделе, квадрат или четырехугольник можно разделить на два треугольника. Это дает нам, что площадь треугольника равна половине площади четырехугольника с тем же основанием и высотой.


    Если площадь параллелограмма A = b ∙ h, то площадь треугольника A = 0,5b ∙ h или

    $$ A = \ frac {1} {2} b \ cdot h $$


    Пример


    $$ \\ A = \ frac {1} {2} b \ cdot h \\\\\\ \: \: \: A = \ frac {1} {2} \ cdot 8 \ cdot 3 \\\ \\\ A = \ frac {1} {2} \ cdot 24 \\\\\\ \, A = 12 \: in ^ {2} \\ $$

    Трапеция — это четырехугольник с двумя параллельными сторонами. {2} $$


    Площадь круга отличается от площади треугольника или четырехугольника.{2} $$


    Видеоурок

    Найдите районы

    Площадь — Площадь трапеций

    Площадь трапеций

    До сих пор мы в значительной степени имели дело с вариациями одной и той же площади A = bh формулы. Мы изменили его на A = lw для прямоугольников и даже на A = s 2 для квадратов, но по сути это то же самое. Сколько раз вы можете перемножить два числа?

    Пора волноваться.Нет, не супер причудливый, а изрядно причудливый. Как иметь две базы — а не две головы — вместо одной.

    Мы говорим ни о чем другом, кроме трапеций , которые действительно легко найти. Пока только две из четырех его сторон параллельны друг другу, мы получаем трапецию. Они даже не обязательно должны быть одинаковой длины, что означает, что другие стороны могут делать все, что захотят, в значительной степени. Совсем ничего похожего на параллелограммы или прямоугольники.

    Мы вам сказали.Странно. Настолько причудливые, что мы должны называть их Freakazoids.

    Во всяком случае, все они выглядят по-разному. Даже если бы мы могли использовать одно уравнение для вычисления площади для всех из них, с чего бы мы начали? Это похоже на попытку попасть из Небраски в страну Оз. (Может быть, это не лучший пример. Объезд через Канзас, и мы золотые.)

    Давайте возьмем общую характеристику всех трапеций: параллельные стороны. Мы знаем, что две параллельные линии всегда будут на одинаковом расстоянии друг от друга, поэтому назовем это расстояние высотой .Для единообразия мы будем использовать х для высоты… снова.

    Мы знаем, что две параллельные стороны не будут иметь одинаковую длину, потому что тогда у нас будет параллелограмм. Поэтому мы назовем эти две стороны основаниями , b 1 и b 2 . (Иногда вы увидите B и b вместо b 1 и b 2 . Это просто разные способы различать две базы.) Наша формула площади для любой трапеции с высотой h и основаниями b 1 и b 2 :

    A = ½ ( b 1 + b 2 b 2 ) h

    К счастью, мы можем игнорировать шаткие стороны, которые не параллельны друг другу.

    Пример задачи

    Если основания трапеции имеют длину 4 и 5 при высоте 2, какова площадь трапеции?

    Используя уравнение для площади трапеции, все, что нам нужно сделать, это указать длину оснований и высоту.

    A = ½ ( b 1 + b 2 ) h
    A = ½ (4 + 5) × 2
    A = ½ × 9 × 2
    9017 A
    = 9 единиц 2

    Пример задачи

    Найдите высоту трапеции, которая имеет площадь 7 единиц 2 и основания 7 единиц и 21 единицу в длину.

    Мы можем использовать ту же формулу площади для трапеции. На этот раз вместо решения для A мы можем решить для высоты h .Замените 21 единицу на b 1 , 7 единиц на b 2 и 7 единиц 2 на A . Вы знаете, что делать.

    A = ½ ( b 1 + b 2 ) h
    7 = ½ (21 + 7) h
    7 = ½ × 28 × h
    0,5 = h

    Высота трапеции 0,5 единицы.

    Какими бы причудливыми ни были трапеции, вы, наверное, уже знаете, что существуют разные их типы. Равнобедренные трапеции пытаются сделать их нормальными, делая две непараллельные стороны одинаковой длины. Правые трапеции имеют сторону, которая образует прямые углы с обоими основаниями.

    Знание этих конкретных типов трапеций не меняет формулы для расчета площади. По-прежнему A = ½ ( b 1 + b 2 ) h . С другой стороны, это может быть полезно, если вам не даны сразу b 1 , b 2 и h .

    Пример задачи

    Какова площадь и периметр этой трапеции?

    Это правая трапеция, поэтому мы можем рассматривать ее как прямоугольник и прямоугольный треугольник, соединенные вместе. Гипотенуза треугольника равна 5, и мы можем найти один из катетов, вычитая маленькое основание из большого, что дает нам длину 3.

    Мы знаем гипотенузу и один из катетов прямоугольного треугольника. . Вы можете сказать: «Теорема Пифагора»? Конечно, можем.Просто замените a на 3, b на h и c на 5.

    a 2 + b 2 = c 2
    3 2 + h 2 = 5 2
    h 2 = 16
    h = 4 единицы

    Высота этой трапеции равна 4, и мы знаем, что ее основания равны 8 и 11. Находим область должна быть ветерок.

    A = ½ ( b 1 + b 2 ) h
    A = ½ (8 + 11) × 4
    A = ½ × 194
    А = 38 штук 2

    Ну, все хорошо, а как насчет периметра? Мы знаем, что периметр складывается из всех сторон.Мы уже знаем три стороны (два основания и неперпендикулярная сторона). К счастью для нас, высота равна одной из сторон для правой трапеции.

    P = b 1 + b 2 + h + s
    P = 8 + 11 + 4 + 5
    P = 28 единиц

    Площадь Калькулятор трапеций 📐

    Быстрая навигация:

    1. Площадь трапеции по формуле
    2. Расчет площади трапеции
    3. Пример: найти площадь трапеции

    Площадь формы трапеции

    Формула площади трапеции: (основание 1 + основание 2) / 2 x высота , как показано на рисунке ниже:

    Расчет в основном основан на том факте, что площадь трапеции может быть приравнена к площади прямоугольника: (основание 1 + основание 2) / 2 фактически является шириной прямоугольника с эквивалентной площадью. Чтобы использовать нашу площадь калькулятора трапеций, вам необходимо выполнить три измерения в одних и тех же единицах (при необходимости преобразовать).

    Результат всегда в используемой единице длины, но в квадрате. Итак, если вы измеряли в футах, результат будет в квадратных футах. Если вы измеряете в сантиметрах, результат будет в квадратных сантиметрах и так далее для квадратных дюймов, ярдов, миль, а также квадратных метров, километров …

    Расчет площади трапеции

    Чтобы произвести расчет, сначала измерьте две базы.Затем постройте высоту, используя прямой угол с кончиком на любом из оснований и используя это основание в качестве одного из его плеч. Измерьте высоту и сделайте необходимые метрические преобразования, пока все три длины не будут в одинаковых единицах измерения. Затем примените формулу выше или используйте нашу область калькулятора трапеций в Интернете, чтобы сэкономить время и повысить вероятность того, что результат будет безошибочным (неправильный ввод обязательно приведет к плохому результату).

    Добавить комментарий

    Ваш адрес email не будет опубликован. Обязательные поля помечены *